You are on page 1of 535

Newton’s Third Law of Motion

Things to keep in mind regarding Newton’s 3rd Law:


➔ Action-Reaction Pair are of same magnitude.
➔ Action-Reaction Pair act in opposite directions.
➔ Action-Reaction Pair act on different bodies.
➔ Action-Reaction Pair act simultaneously
➔ Action-Reaction Pair are of same nature.
Basic Forces
Weight of the body

Normal force

Tension force
Weight Force
Weight
. of the body

It is a Gravitational force
Magnitude = mg
(m = mass of body, g = acceleration due to gravity)

It is experienced by any massive body due to Earth’s gravitational attraction


Direction is towards the centre of the earth or vertically downward.
Mark the Force of Gravity m

m
Mark the Force of Gravity m

m
mg

mg

mg
m

Each & every block experiences mg vertically downwards.


mg
Normal Force

N
Normal Force
It comes into action whenever 2 bodies come in contact with each other
Electromagnetic force.
It is the interaction force between the charged particles of the two surfaces in
contact.

MAGNITUDE
It is a self adjusting force & the magnitude is dependent on the situation.

DIRECTION
It is always perpendicular to the common surface in contact with each other &
away from the body (pushing).
Mark the Normal Reaction

m
Illustration
Normal force on block due
N to surface. (Action force)

N
Normal force on surface
due to block. (Reaction force)
Illustration
Normal force on block due
to surface. (Action force)

Normal force on surface due to


block.(Reaction force)
Mark the Normal Reaction
Mark the Normal Reaction

Horizontal & Vertical plane applies Normal Force


on the rod perpendicular to contact surface.
Mark the Normal Reaction
Mark the Normal Reaction
Mark the Normal Reaction

NOTE:- For cylinder, disk, ring, sphere normal force will always pass through
the center of the structure.
Mark the Normal Reaction
Mark the Normal Reaction

NOTE:- For cylinder, disk, ring, sphere normal force will always pass through
the center of the structure.
Tension force
Tension force

Tension force in string


It is an Electromagnetic force
This force comes into action, when the string is stretched/ taut.
It is the interaction force between the charged particles of a string.
It is always along the length of the string and is PULLING by nature.
Tension force

Magnitude
Can take any value depending upon the question or strength of string.

Direction
Along the length of string and away from body
Types of Strings
How many types of Strings are there ?

T T

Strings are of 2 types

Ideal string Non–Ideal string


Ideal string Non–Ideal string

String is assumed to be massless String has some appreciable mass

Tension throughout the length of Tension in the string is different at


string remains CONSTANT DIFFERENT POINTS

NOTE:- The string is said to be INEXTENSIBLE (i.e length does not change)
Illustration
➔Block of mass m is attached with the IDEAL string.
C Now as the block experiences mg vertically downward
➔It basically stretches the string and tension force is
B generated. Now as the string is ideal (i.e massless),
tension force
A
➔Throughout the string will be the same
m
What is direction of tension force ?

Answer: It depends on what is being observed


C

m
Illustration
➔ If we observe block, as the block is
pulling the string down, hence tension at T
point A on block is upwards. A

➔ However if we observe string then M


T
tension is vertically downward at point A.
➔ In similar fashion let us find the tension
force at point C
Tension Tension
NOTE:- Block of mass M is attached at C at C
with the IDEAL string. on the on the
string celing
C

B
T
A

Tension at C Tension at C
on the string on the ceiling
Draw the Tension Forces

m
Draw the Tension Forces

T1
T2

m
Draw the Tension Forces

m
m
Draw the Tension Forces

m1

m2
➔Blocks of mass m1 & m2 are attached with the
strings. T1

➔Tension on block of mass m1 due to both the


strings will be along the string & away from the T1
block.
m1
➔Tension on block of mass m2 will be along the
string & away from the block. T2

T2

m2
Newton’s Laws of Motion
LECTURE 2
Free Body Diagram

mg
Free Body Diagram
F.B.D

It is pictorial representation of all the forces acting on a body.


STEPS to draw F.B.D.
1) Identify the body (or system) which is to be analyzed
2) Choose the co-ordinate axis as per convenience
3) Represent all the forces ACTING on the body by vectors

NOTE: Forces exerted by the body should not be drawn Include only
those forces which are acting on the body and not those forces which
the body itself is exerting
Draw F.B.D. of block of mass ‘m’

m
Now focus on which forces have we included & the Action Reaction Pairs

m
Now focus on which forces have we included.

This force is acting ON


m
Action force lock hence we include it
from IIIrd law

mg
This force is
applied by block
hence we DON’T Reaction force
INCLUDE it mg
Similarly

Action force
N

Reaction force
From IIIrd law
Finally N and mg are NOT action reaction forces
Reason : N and mg are of DIFFERENT
nature
N

mg
Now let’s draw the FBD of each block

m1

m2
Imagine blocks M1 and M2 kept on ground as

FBD of M1 FBD of M2
Normal Contact force
N1 Normal Contact force N2 on m2 due to ground
on m1 due to m2
m m2
1
m1g Gravitational force N1 m g
Gravitational force on m2
2
on m1

NOTE:- Normal Contact force on M2due to M1


Do not draw M1g directly
What is a System?
System
System is a body or combination of bodies which are being observed.

Man

Monkey
Horse
➔Imagine these 3 beings standing on the
Man
surface of earth.
➔We can assume any one of the body to be
our system

Monkey
Horse
Now the question is whether is this the only System?

Man

Monkey
Horse
What exactly is the system in the diagram?
The answer is clear that it is just a
matter of choice i.e. what actually is
to be observed becomes our system

We can assume m1 to be our system , Or if required as per the question we can


assume m2 to be our system, Or m1 , m2 , string , pulley, table & earth can all
combine to form a system.
Classification of Forces

Internal External
If the force acting on a body/particle is
Internal
applied by some other body which is
included in the system, then such a force is
termed as Internal force.
FORCE
If the force acting on a body/particle is
External applied by some other body which is not a
part of the system & acts as an external
agent.

Note: However the application of the same will be discussed later in the chapter.
Ram Shyam

Imagine Ram and Shyam playing tug of war.


If we consider Ram, Shyam & string to be a
system, then the pulling force acting on
Ram due to Shyam (part of system).
Or the pulling force acting on Shyam due to
Ram (part of system).
Both are the examples of INTERNAL Force
Ram Shyam

The other classification is we consider only


Ram to be our system
And the force experienced by Ram due to
Shyam is said to EXTERNAL FORCE.
Hence Shyam acts as an external agent.
Concept of Weighing Machine
Concept of Weighing Machine

m
Weighing Machine
Weighing Machine

m
Concept of Weighing Machine

m
Weighing Machine
Weighing Machine

N
N

N – mg = m (0)
m
⇒ N = mg

mg
N

m
Weighing Machine

mg
N

N
Reading of weighing machine = kg.
g

=
mg
kg N = mg
g As calculated
.
earlier
= m kg.
N
+

m
– Weighing Machine

mg

N
Reading of weighing machine - Summary

N – mg = m (0)
➔ Normal force (N) will be vertically upward
m ➔ Apply all the forces on block of mass m
⇒ N = mg ➔ Weight mg will be vertically downward
➔ Acceleration of block, a = 0
mg ➔ Take upward direction as +ve and
downward direction as –ve.
➔ So, N will be +ve and mg will be -ve
Concept of Weighing Machine N

m m
Weighing Machine
Weighing Machine

mg
N

Weighing machine is sensitive for NORMAL FORCE it applies on body.


N kg
Reading of weighing machine (in kg) = g
.

Where,
N = Normal force applied by weighing machine on body (in Newton)
g = acceleration due to gravity (in m/s2)
Example A block of mass m is kept on weighing machine, which is placed in lift,
accelerating up with acceleration ‘a’. Find reading of weighing machine.

A. (mg - ma)/g
B. (mg + ma)/g
C. m
D. None of these a

m
Weighing Machine
Example A block of mass m is kept on weighing machine, which is placed in lift,
accelerating up with acceleration ‘a’. Find reading of weighing machine.

Solution:
N Normal force (N) will be
N vertically upward

a Applying Newton’s 2nd law on


block.

m ⇒ N – mg = ma
Weighing Machine
m
+ a ⇒ N = mg + ma

mg
– mg

Ground
observer
Example A block of mass m is kept on weighing machine, which is placed in lift,
accelerating up with acceleration ‘a’. Find reading of weighing machine.

Solution:
N
∴ Reading = g kg.

=
mg + ma
kg.
g

⇒ N = mg + ma

1) Apply all the forces on block of mass m


2) Consider an observer on ground (INERTIAL FRAME) For observer,
Acceleration of block = a
3) Take upward direction as +ve and downward as –ve. So N will be +ve and mg
will be –ve
2 blocks of mass 2kg and 4kg are kept on a smooth floor as shown.
Example Find the normal contact force
I. between the blocks
II. between 4kg and surface.
A. 20 N, 60 N
2 kg
B. 60 N, 20 N
C. 40 N, 60 N 4 kg
D. None of these
Example Two blocks of mass 2kg and 4kg are kept on a surface as shown. Find
the normal contact force
I. between the blocks
II. between 4kg and surface.
Solution:
N1 Applying IInd law 2 kg
+
4 kg
2 kg Fnet = M a
N1 – 20 = M (0)
20 N
N1 = 20 N

Let us start with FBD of the blocks


Two blocks of mass 2kg and 4kg are kept on a surface as shown. Find
Example
the normal contact force
I. between the blocks
II. between 4kg and surface.

Solution: N2 Applying IInd law 2 kg

Fne = Ma 4 kg
4 kg
t
N2 – N1 – 40 = M (0)
N1 40 N
N2 = N1 + 40
N2 = 20 + 40
N2 = 60
N
From FBD of 2kg we Calculated
N1 = 20N

N2 Normal contact force between 4kg and ground


N1 Normal contact force between 2kg and 4kg
Example If force F = 5N, then find :
a) Acceleration of 2 blocks.
b) Contact force between m1 and m2.

A. 5/3m/s2 , 23/7 N
B. 5/4m/s2 , 24/7 N
4 kg
F = 5N 3 kg
C. 5/7m/s2 , 20/7 N
m2
D. None of these m1
4 kg
F = 5N 3 kg
m2
m1
Solution: a

F m1 m2

➔We have seen, both block moved together.


➔Take both blocks as a system.
➔So, acceleration of both block will be same.
Acceleration of system = a (let)

Mass of system = m1 + m2 = 3 + 4 = 7kg


Force on system = F = 5N
Force on system
∴ Acceleration of System =
Mass of system
a = 5/7 m/s2
Solution:
a

F m1 m2 N
N

+
Applying Newton’s 2nd law on block m2.
N = m2a
5
As, m2 = 4kg and a = m/s2
7

5 20
⇒ N = 4× ⇒ N = N
7 7

1. Let’s apply contact forces between two blocks


2. To find contact force, take m2 as the system
3. Apply Newton’s 2nd law on block m2 in horizontal direction
4. Contact force (N) on m2 will be in right direction and on m1 in left direction
Motion on Inclined Plane
A block of mass m is placed on an inclined plane of inclination θ with the horizontal. If
Example all the surfaces are smooth and the inclined plane is fixed on ground. Find the
acceleration of block and the normal contact force between the block and wedge just
after the block is released.

A. g sinθ, mg cosθ
B. g cosθ, mg cosθ
C. g sinθ, mg sin θ
D. None of these

θ
fixed
Solution:
➔ As the wedge is fixed on ground.
➔ The block is constrained to move only
along the wedge Or we can say the
acceleration of block in the direction
perpendicular To wedge is zero.
➔ Take along the inclined & downward
direction as +ve and upward as –ve
➔ Take perpendicular to inclined & upward
direction as +ve and downward as –ve θ
➔ Now apply IInd law fixed
N
Solution:

Along the incline Perpendicular to incline


Fnet = Max Fnet = May
θ
As ay = 0
Mg sinθ = Ma
Fnet = 0 mg cos θ
θ
a = g sin θ N = mg cos θ fixed

Concept : Once the motion of particle is clear always choose co–ordinate axis
parallel and perpendicular to that direction.
String based Questions
Example Block of mass M is attached with ideal string (negligible mass). Find tension at
(i) point A
(ii) point B (Point B is midpoint of string).

A. Mg, 0
B. 0, Mg
C. Mg, Mg
D. None of these B

M
Solution:
Tension at point A :
Tension at point A :
T
Applying Newton’s 2nd law
F = Ma B
M
⇒ TA – Mg = M(0) = 0 +
A
(acceleration of block = 0)
M M
g
So, TA = Mg –
Solution:
Tension at point B:
Let's consider block & string below point B as the body.

So, Tension will be same throughout the length of string. + B TB

A
Tension TB on body will be along the string & away from body. M

Applying Newton’s 2nd law –


Mg

F = Ma

⇒ TB – Mg = M(0) = 0

(acceleration of block = 0)

So, TB = Mg

Concept: To find tension at any point to break the system at that point
Two blocks of mass 4kg and 6kg are attached in a vertical plane
Example with the help of ideal strings. Find the tension at points
I. A
II. B A
A. 100N, 60N
B. 60N, 100N
4kg
C. 80N, 60N
B
D. None of these

6kg
Solution:
1. Let us draw FBD of the two blocks
2. Applying newton’s IInd law
T1 T2


T2 40N 60N

4kg 6kg

= Ma Fnet = Ma
Fnet
As = 0 As a = 0

Fa net = 0 Fnet = 0

T1 – T2 – 40 = 0 ….(1) T2 – 60 = 0 ….(2)
Solution:
Now as the string 1 and 2 are ideal hence the
T1 – T2 – 40 = 0 ….(1) tension throughout the string will be constant

T2 – 60 = 0 ….(2)

T2 = 60N
A
Putting this value of T2 in equation (1)
String 1
T1 – T2 – 40 = 0
4kg
T1 = 100N B

Tensions at A = T1 = 100N String 2


Tension at B = T2 = 60N
6kg
Example A block of mass M is suspended with the help of a massive string of mass 2m
as shown in the figure. Find the tension in the string at point
i) B (lower end)
ii) A (midpoint)

M
Solution:
T

As the acceleration of the block is zero, these


Mg forces should add to zero. A
The force on it are
Take the block as the system.
(a) Pull of the string, T, upward, B
(b) Pull of the earth, Mg, downward, M
T - Mg = 0
T = Mg
Solution:
To get the tension T at A we need the force exerted by the
upper part of the string on the lower part of the string. Take
the string below A as the system. The forces acting on this part
are
a) T′ upward by the upper part of the string
b) mg downward by the earth
c) T downward by the block.

We have already used the symbol T for the force by the string
on the block.

Applying Newton's First Law gives


T′ = T + mg = (M + m) g
Example Breaking Tension of string = 200N. Find the maximum acceleration of boy
without breaking the string. (mass of boy = 10 kg )
A. 10 m/s2
B. 5 m/s2
C. 2 m/s2
D. None of these
Solution: Let acceleration of boy is a upward
Applying Newton’s 2nd law on boy.

Maximum tension provided by string is also


called breaking tension. For maximum
acceleration, tension should be maximum.
T

mg
Solution:
Applying Newton’s 2nd law on boy
T – mg = ma
T– mg
⇒ a=
m T
⇒ a = 200 – 10(10) a
10
100
⇒ a = 200 – 100 =
10 10
mg
a = 10m/s2
Example Breaking tension of string is 90N. Find the maximum acceleration of
boy with which he can climb up without breaking the string.
(mass of boy = 10kg)
A. -1 m/s2
B. -2 m/s2
C. 1 m/s2
D. boy can't climb up
Solution:

Applying Newton’s 2nd law on boy


T – mg = ma
T– mg
⇒ a= m T
⇒ a = 90 – 10(10) a
10
90 – 100 -10
a
⇒ = = = –1
10 10
mg
As acceleration is –ve even at maximum
tension, boy cannot climb up.
Newton’s Laws of Motion
LECTURE 3
String & Pulley Problems

m2

θ
Example Find acceleration of blocks, also find tension in string 1 & 2

2 1 F = 120N
3kg 4kg 5kg
2 1 F = 120N ALTERNATIVE
3kg 4kg 5kg
Solution:
(i) For 4 kg block (A) T1
Fnet = Ma 4 kg

… (i) a A
T1 – T2 – 40 = 4 a T2
3 kg
(ii) For 3 kg block (B) +
T2 40 N a B
Fnet = Ma
T3 –
T2 – T3 – 30 = 3 a ... (ii) T3 30 N
3 kg
(iii) For 3 kg block (C) a C

Fnet = Ma
T3 – 30 = 3 a ... (iii) 30 N
Example Find acceleration of blocks, also find tension in string 1 & 2

2 1
3kg 4kg 5kg F = 120N

Solution:
Mass of system = 12 kg As all three blocks
Force on system = 120 N are connected with
strings. So all three
∴ Acceleration of = Force on system blocks will move
System Mass of system with same
120 = 10 m/s2 acceleration (a).
=
12
Solution:
a = 10m/s2
(2) T2 (1) T1
3kg 4kg 5kg F = 120N

Applying Newton’s 2nd law on system


T1 = ma
⇒ T1 = 7 × 10 ⇒ T1 = 70N
Applying Newton’s
2nd law on 3kg block
T2 = ma
⇒ T2 = 3 × 10 ⇒ T2 = 30N 1. To find tension in string 1, blocks 3kg and 4kg will be
considered as System. In horizontal direction, tension
T1 is the force on the system
2. To find tension in string 2, block 3kg will be
considered as body. In horizontal direction, tension T2
is the force on 3kg
3 blocks of mass 4 kg, 3 kg and 3 kg are attached to ideal strings. If a force of
Example 150 N is applied at point A in vertically upward direction then find the tension in
the strings.
A. 150N, 90N, 45N
150 N
B. 90N, 90N, 45N
C. 90N, 130N, 45N A
1
D. 150N, 80N, 45N

4 kg

3 kg

3 kg
ALTERNATIVE

150 N

A
1

4 kg

3 kg

3 kg
Solution:

T1 T T
4 kg 23 kg 3 3 kg
a A a B a C

T2 40 N T3 30 N 30 N

1. Let us start with FBD of each block.


2. Let us assume acceleration of block A to be ‘a’ in vertically upward
direction.
3. Since the string is inextensible from constrained motion we can say that
each block will have the same acceleration.
Solution:
T1 – T2 – 40 = 4 a

T2 – T3 – 30 = 3 a
… (i)
T3 – 30 = 3 a

T1 – 40 – 30 – 30 = 10 a

T1 = 100 + 10 a

∴ T1 = 150 N
... (ii) • Adding (i). (ii), and (iii)
• As the strings are ideal, hence tension is through out
Hence we get
same
150 = 100 + 10a

a = 5 m/s2 ... (iii)


Solution: T1 – T2 – 40 = 4a …(i)
a=5 m/s2 T2 – T3 – 30 = 3a …(ii)
Putting this value of a in each equation
Eq.
(1) T2 = T1 – 40 – 4a
⇒ 150 – 40 – 20
T2 ⇒ 90 N

Eq. (2)
T3 = T2 – 30 – 3a
⇒ 90 – 30 – 15
T3 ⇒ 45 N
A groove OA = 5 m cut in the plane makes an angle 30° with OX. A ball is free to slide
Example down in the influence of gravity. Find the time taken by the cylinder to reach from A
to O. (g = 10 m/s2)

A. 4 sec
B. 3 sec
C. 2 sec
D. 1 sec
A

30°

30°
O x
A

30°

30°
O x
Solution:

A
30°
30° x
O
Looking at the incline plane from
left, we get the image as:

mg cos30°
30°
mg

Resolving the force ‘mg’ along the plane


and perpendicular to the plane.
Solution:
Let’s view the force on the cylinder along the plane
Resolving the force ‘mg sinθ’ along the groove
A
Hence, net force on cylinder along the grove
= mg sin30° cos60° mg sin30° 30°
x
30°
mgsin30°cos60° O
∴ Acceleration along the
groove = m
1 1
= g× 2 2 ×

g
=
4
Displacement along the groove = 5m (=AO)
Solution:
1
s = ut + 2 at2
and, u = 0
1 2
∴s = at
2
1 × g 2
⇒5 =
2 4 t
5×8 = 4
⇒ t2 =
10

⇒ t = 2 sec

∴ Time taken = 2 sec


One end of a string 0.5 m long is fixed to a point. The free end has a downward
Example force of 8 N and a horizontal force F acting on it, until it is 0.3 m from the
vertical through A. Find the tension T in the string & the force F.

A. 10 N, 6 N
B. 8 N, 6 N
fixed
C. 6 N, 10 N
D. 6 N, 8 N

0.3 m
F

8N
Solution:

Given, AC = 0.5 m, BC = 0.3 m A

∴ AB = 0.4 m (By Pythagoras's Theorem) T


θ θ
and if ∠BAC = θ
B F
C
AB 0.4 4
Then cos θ = = =
AC 0.5 5
,
BC 0.3 3 8N
and sin θ = = =
AC 0.5 5
Solution:

Resolving the tension force in horizontal and A T cosθ

vertical direction.
Applying Newton’s First law in Vertical T
θ
direction:
θ
B F
T cosθ = 8 N
T sinθ
4
or, T =8N
5
or, T = 10 N 8N

Applying Newton’s First law in Horizontal


direction:

F = T sinθ
3 F = 6N
F = 10 5 N
Example Determine the tensions T1 and T2 in the strings as shown in figure.

A. 150 N, 120 N
37°
B. 120 N, 150 N
T1
C. 90 N, 0 N
D. None of these
T2

9 kg
Solution:

Resolving the tension T1 along horizontal and


vertical directions.
T1 sin37° 37°
Applying Newton’s First law in Vertical direction:
T1
37° T cos37°
T1 sin 37° = 9 × 10 N ….(i) T2
1

Applying Newton’s First law in Horizontal direction :

T1 cos 37° = T2 ….(ii)


Solution:
T1 sin60° 60°
T1
T1 sin 37° = 90 N ….(i) 60°
T1 cos60°
T2
T1 cos 37° = T2 ….(ii)

From equation (i) From equation (ii)

T1= 150 N T2 = T1 cos 37°

= 120 N
String & Pulley based Questions
Example Consider the system of ideal pulley as shown. A block of mass m kg is attached
with the help of ideal string. Find the Force exerted by pulley on the ceiling.

A. mg
B. 2mg
C. 0
D. None of these

m
Solution:
Draw FBD

Block Pulley
+ T T′
+

mg
T T

Applying IInd law Applying IInd law


Fnet = Ma Fnet = Ma
T – mg = M(0) T′ – 2T = Mpap
T = mg T′ – 2T = 0

T′ = 2mg
Example Find acceleration of the blocks & tension in the string (m2 > m1).

m1 m2
m1 m2
Solution:

1. For mass m1,Take upward


direction as +ve & •
downward direction as – + –
T
ve. So T will be +ve & m1g T

will be –ve.
2. For mass m2,Take a
a m1 m2
downward direction as
– +
+ve & upward direction
as –ve. So T will be –ve & m2g
m 1g
m2g will be +ve.
Solution:

Applying Newton’s 2nd law on mass m1 •


+ –
T – m1 g = m1 a … (i) T T

Applying Newton’s 2nd law on mass m2


a m2 a
m1
m2g -T = m2a … (ii) – +

m1g m2g
Solution:
Adding equations (i) and (ii)
T – m1g = m1a … (i)
m2g -T = m2a … (ii)

m2g –m1g = m1a + m2a


⇒ (m2 – m1)g = (m1 + m2)a

m2 – m1
∴ a = g
m2 + m1

Putting value of ‘a’ in either of Eqn. (i) or (ii), we can get value of T.

2m1m2
T = g
m2 + m1
Example 2 masses m1 and m2 are connected by a light string going over a smooth light
pulley as shown.The system is at rest. Find the angle of the incline & force
exerted by the incline on m1

m2

θ
Solution:
Take the body of mass m2 as the system.
The forces acting on it are :
m2
(i) m2g vertically downward (by the earth),
(ii) T vertically upward (by the string),
As the system is at rest, these forces θ
should add to zero.

This gives T = m2g . . . (i) T

m2

m2 g
Solution:
Next, consider the body of mass m1 as the system.
The forces acting on this system are:
(i) m1g vertically downward (by the earth), m2

(ii) T along the string up the incline (by the string.)


(iii) N normal to the incline (by the incline.) θ
Taking components parallel to the incline,
T = m1g sin θ . . . (ii)

N T
➔As the string and the pulley are all light and
smooth, the tension in the string is uniform m1g sinθ
everywhere. m1g
➔As the system is in equilibrium, these forces
should add to zero.
Solution:
Taking components along the normal to the
incline, m2
N = m1g cosθ . . . (iii)

Eliminating T from (i) and (ii)


θ
⇒ m2g = m1g sinθ.
⇒ sinθ = m2/m1
⇒ θ = sin-1 (m2/m1)
N T
N = m1 g 1-(m2/m1)2
m1g sinθ m1g cosθ
m1g
Newton’s Laws of Motion
LECTURE 4
Constrained Motion - 1

A

B 3 m/s2
Constrained Motion
A motion in which motion of all the bodies
in a system are depended on each other
Constrained Motion

2 m/s

600

➔ Mathematical relation by which motion of bodies in a system are related is called constrained relation.
➔ By motion, we imply displacement ,velocity ,acceleration.
Constrained Motion
SA SB

A B

Block A and B are connected with an inextensible string

Hence, constrained relation is


Velocity & Acceleration of both bodies will be equal
Constrained Motion
y x

A B

Block A and B are connected with an inextensible string

So, displacement of B = displacement of A


x=y
Differentiate w.r.t. time: VB = VA Velocity and Hence, constrained relation is
Acceleration of both
Differentiate w.r.t. time: aB = aA bodies will be equal
sA = sB
VA = VB
So, the relations between displacement /
velocity of A, B are called constrained relation. aA = aB
Example Particle B moves along the horizontal path with speed VB.
Then find the speed of A at an instant when the string at B makes an angle θ
with the horizontal.
A. VA = VB cos θ
B. VA = VB sin θ
C. VA = VB cot θ
D. None of these

A
θ
B VB
ALTERNATIVE

A
θ
B VB
Solution:
Component of velocities / accelerations along the length of string /rod
should always be same. (As the length of the string does not change).
Let us resolve the velocities of two ends into two
Components. One along the length and the other
perpendicular to the length. We clearly see VB sin θ
being perpendicular to the string does not change
the length of string.
And as the length of the string remains constant

VA

VB sin θ VA = VB cos θ
A
θ
B VB
θ

VB cos θ
Constrained Motion based Problems

Pulley-Block Constrained Motion


Method 1: Segment method
Concept: As the string is inextensible, elongation in the string is Zero
x2 x1

P String Q

Elongation in string
(+ x1) + (–x2) = x1 – x2
As elongation in string should be zero.

x1 –x2 = 0 ⇒ x 1 = x2
Method 1: Segment method
Concept: As the string is inextensible, elongation in the string is Zero
x2 x1

P String Q

Elongation in string
(+ x1) + (–x2) = x1 – x2
As elongation in string should be zero.

x1 –x2 = 0 ⇒ x1 = x2 Differentiate w.r.t. time:v = v x1 = x2


1 2
v1 = v2
Differentiate w.r.t. time: a1 = a2
a1 = a2

1. Consider a string PQ. Point Q is displaced by x1 and point P is displaced by x2


2. Any displacement towards the center of the string will be considered NEGATIVE
3. Any displacement away from the center will be considered as POSITIVE
Let’s take an example of Atwood Machine…

m1 m2
Q R
➔ String has two segments PQ and RS.
x1
➔ Displacement of any point of segment
attached to fixed pulley will always be
P S
A considered to be zero.eg. point Q and R
B x2 ➔ Let displacement of block A in upward
direction = x1
So, total elongation = (–x1) + (+x2) = 0 ➔ Displacement of block B in downward
direction = x2
➔ In segment PQ, end P is going towards
Differentiate w.r.t. time: v1 = v2
center so it’s contribution to elongation is (–
Differentiate w.r.t. time: a1 = a2 x1)
➔ In segment RS, end S is going away from
center so it’s contribution to elongation is
(+x2)
Increase Decrease
• •

v
?

• •
Method 2: Tension Trick
Net work done by massless string on blocks will
be ZERO.

→ →
∑T . x = 0

On Differentiation.
→ → T T
∑ T . v = 0
x
On Differentiation. A B x
→ →
∑ T . a = 0
Short Trick:

x1 T
A T
B x2

→ → → →
∑ T. x =0 T . x = Tx cos π
Tx1 +( – Tx2) =0 ⇒ –Tx

Tx1 = Tx2

x1 = x 2
On Differentiation v1 = v 2 As from block A to B there is single and massless
On Differentiation a1 = a 2 string, tension throughout the string will be same
Pulley Trick

a1

a0

a2
Constrained Motion Examples
If block B moves downward with acceleration 3 m/s2, find
Example acceleration of block A.

A. 2 m/s2
B. 4 m/s2
C. 6 m/s2
D. 8 m/s2


A

B 3 m/s2
ALTERNATIVE


A

B 3 m/s2
Solution: a1
T
2T
A •

∑ → . →

= 0 T
T a
2T
⇒ Ta1 – 2Ta2 = 0
B a2
⇒ Ta1 = 2Ta2
⇒ a1 = 2a2
⇒ a1 = 2 × 3 = 6 m/s2
Example If velocity of block A is 2m/s upwards, find velocity of block B?
A. 4 m/s
B. 8 m/s

C. 16 m/s
2 m/s
D. None of these

A •

B v
ALTERNATIVE

2 m/s

A •

B
v
Solution: •
8T 8T
∑ → . → = 0 v1
T v A •
4T 4T
⇒ 8Tv1 – Tv2 = 0 •
⇒ 8Tv = Tv 2T 2T
1 2

⇒ v2 = 8v1
T T
⇒ v2 = 8 × 2 = 16 v2
B
m/s
Example In the system shown in figure find the speed of block C?
A. 4 m/s
B. 8 m/s
C. 16 m/s • •
D. 32 m/s

• •

A B
C
v
3 m/s
5 m/s
ALTERNATIVE

• •

• •

A B
3 m/s C
5 m/s
v
Solution:

Applying Short Trick


• •
→ → T T T T
∑ T. v = 0
– 2 T (vA) – 2T (vB) + T (vC) = 0 2T 2T T
VC
⇒ – 2 vA – 2vB + vC = 0 A B C
⇒ – 2 (3) – 2 (5) + vC = 0
VA VB
⇒ vC = 16 m/s
Newton’s Laws of Motion
LECTURE 5
Constrained Motion - 2
In the system shown, the block A moves towards left at acceleration of 3 m/s2.
Example Find acceleration of rod B which is constrained to move vertically over the
wedge. θ = 30o
A. √3 m/s2
B. 3 m/s2
C. 1/√3 m/s2
D. 1 m/s2 B

aA= 3 m/s2

A
θ
ALTERNATIVE
B

A A
θ θ
x
ALTERNATIVE - 2

aA= 3 m/s2

A
θ
Solution:

Concept : The components of


B
acceleration of the two bodies along
the common normal are equal

➔Let Acceleration of A is aA aA= 3 m/s2


leftwards and acceleration of
A
B is aB downwards.
θ
➔Let’s draw components of
acceleration of A, B along
perpendicular to the surface aA
of incline of A.
90–θ
θ

aB
Solution:
Components of acceleration of both A and B
B
along perpendicular to surface of wedge A aA= 3 m/s2
should be same.
A θ
∴ aA sin θ = aB cos θ
⇒ aB = aA tan θ = aA tan 300 =

aA
90–θ
θ

aB
Example As shown this rod’s lower end A is pulled towards left with a constant
velocity v. Find the velocity of the other end B downward when rod makes an
angle θ with the horizontal.

A. v tanθ B
B. v cotθ
C. v cosθ
D. None of these

θ v
A
ALTERNATIVE

θ v
A
Solution:

➔Thus the two points must have same


B
velocity components in the direction of
line joining them:
➔In cases when relation between two 90–θ
u
points of a rigid body is required, we can
v v
make use of the fact that in a rigid body A
θ

the distance between two points always θ

remains same.

∴ usin θ = v cos θ
cos θ
⇒u=v
sin θ
⇒ u = v cot θ
In figure below, the mass of the ball is η times as great as that of the rod. The
Example length of the rod is L, the pulleys are massless, as well as the friction are
negligible. The ball is set on the same level as the lower end of the rod & then
released. How soon will the ball be opposite to the upper end of the rod?
L(η+4)
g(2−η)

L(η+4)
3g(2−η)
2L(η+4)
3g(2−η) •

L
m

ηm
Solution:

L
m

ηm
In the arrangement shown in figure below, the mass of the ball is η times
Example as great as that of the rod. The length of the rod is L, the masses of the
pulleys and the threads, as well as the friction are negligible. The ball is
set on the same level as the lower end of the rod and then released. How
soon will the ball be opposite to the upper end of the rod?


➔ Hint: From constraint relation we can see
that the acceleration of the rod is double
than that of the acceleration of the ball.
➔ If ball is going up with an acceleration a,
rod will be coming down with the •
acceleration 2a.

L
m

ηm
Solution:
The relative acceleration of the ball with respect
to rod is 3a in upward direction. If it takes time t
seconds to reach the upper end of the rod , we
have
. . . (i)

Let mass of ball be m and that of rod is M,


the dynamic equations of these are

For rod Mg − T = M(2a) . . . (ii)


For ball 2T− mg = m(a) . . . (iii)
Solution:

Substituting m = ηM
and solving Eqs. (ii) and (iii), we get

a = 2−η g
η+4

From Eq. (i), we have 2L(η+4)


t= 3g(2−η)
Example A block of mass m is resting on a wedge of angle θ as shown. With what minimum
acceleration ‘a’ should the wedge move so that the mass m falls freely?

A. g
B. g cosθ
C. g cotθ
D. g sinθ
a

θ
Solution:

Similarity

a
N
Solution: ma sinθ
a
ma(pseudo force)
Resolving the force ‘mg’ along the plane and ma cosθ
mg sinθ
perpendicular to the plane. mg cosθ
mg θ
Resolving the force ‘ma’ along the plane and
perpendicular to the plane
Balancing the forces perpendicular to the plane:
N + ma sin θ = mg cos θ
The block will fall freely if N = 0
i.e., ma sin θ = mg cos θ

⇒ a = g cot θ

∴ Option (c) is correct.


Solution:

For ground observer N


a

mg cos θ – N mg sinθ
ablock ⊥ = mg cosθ
mg θ
m

N=0
θ
ablock ⊥ = g cos θ a

a sin θ θ
a sin θ ≥ g cos θ
a ≥ g cot θ

⇒ a = g cot θ
One body moves on another moving body Wedge A is moving with acceleration a
Example as shown in figure. Find acceleration of block B w.r.t. : (i) Wedge A
A. a
B. a sinθ
C. a cosθ
D. a cotθ

a
A
θ
P aA Q
Solution:
R aB/A
i) Acceleration of block B w.r.t. Wedge A B
a aB/A
➔ There are two segments of string PQ and QR
A
➔ As wedge A is accelerating with a left wards, so θ
with the respect to wedge A point P will appear
to accelerate with a right wards.
➔ Let block B is accelerating downwards with aBA
along the surface of wedge A.
➔ In segment QR aBA is away from centre of
segment, its contribution to elongation will be +
aBA
➔ So point R which is attached with block B, will
also accelerate with aBA downwards.
➔ In segment PQ a is towards centre of segment,
its contribution to elongation will be – a
Solution: P aA Q

R aB/A
i) Acceleration of block B w.r.t. Wedge A
a B
Total elongation in string = 0 aB/A
A
∴ –a+a BA = 0 θ

⇒ aBA = aA
Example One body moves on another moving body Wedge A is moving with acceleration a
as shown in figure. Find acceleration of block B w.r.t. : (ii) Ground

a
A
θ
a
A
θ
Newton’s Laws of Motion
LECTURE 6
Pseudo Force

a0
θ

m
Inertial frame of reference
Any frame/observer which is either stationary or moving with uniform velocity

m
v

Consider a passenger inside a car moving with CONSTANT


VELOCITY & a person standing on the ground is observing the car.

As we know ground is frame of reference.


➔ Hence the Person standing on the ground is also Inertial Frame
of reference. Now as the car is moving with constant velocity.
➔ So car is Inertial Frame of reference.
➔ the driver of the car is observing from Inertial Frame of
reference.
Inertial & Non-Inertial frame of reference
Any frame/observer which is accelerating with respect to an inertial frame
is called non-inertial frame

a
θ

m
a

In this case, the car is Accelerating.


And a person standing on the ground is observing the car.

So, car is the NON-INERTIAL frame of reference. And so, the


person inside the car is observing from a NON-INERTIAL frame.

UNACCELERATED Observer ⇒ INERTIAL FRAME


ACCELERATED Observer ⇒ NON-INERTIAL FRAME
Pseudo Force
Uniform Velocity
m

m
Uniform Velocity
v m

➔ Let’s take a situation to understand the concept of pseudo force


➔ Now observe the block from car moving with uniform velocity
➔ From car, it will appear that block is moving backwards with uniform velocity
➔ As car is moving with uniform velocity, car is inertial frame
➔ So Newton’s laws are valid for inertial frame also
Acceleration
m or

m
Acceleration
a ma m
m (pseudo force)
or

➔ Let’s observe the block from accelerated car (non-inertial frame)


➔ From accelerated car, block will appear to accelerate in backward direction
➔ But there is no force in backward direction
➔ So, from non-inertial frame Newton’s laws are not valid
➔ To validate Newton’s laws from non-inertial frame we need to introduce concept
of pseudo force
Pseudo Force
In order to apply Newton’s laws in non- inertial frame, we need to apply one extra force apart from
other real forces. This force we call Pseudo force

Magnitude

Pseudo force = Mass of body × Acceleration of non- inertial frame

Direction
Opposite to direction of acceleration of non- inertial frame.
B

a
m
Illustration: i) Lift is accelerating upward:

Let’s find the tension in the string from non-inertial frame

a m
Illustration:
i) Lift is accelerating upward: ➔ Let’s find the tension in the string
from non-inertial frame
T +
➔ Lift is accelerating upwards so lift
m a is non-inertial frame
A ➔ Imagine an observer A inside lift
mg –
as observer is in accelerated
ma frame so we need to apply pseudo
Applying Newton’s 2nd law on block. force

T – mg – ma = m(0)

T – mg – ma = 0

T = mg + ma
i) Lift is accelerating upward:
T +

m a

mg –

Applying Newton’s 2nd law on block. B

T – mg = ma

T = mg + ma
➔ Consider an observer B on ground
(inertial frame)
➔ Let’s find tension in string from
inertial frame
➔ For observer B, acceleration of
block will be a upwards
PSEUDO FORCE

Pseudo Force Problems


A pendulum is hanging from the ceiling of a car having an acceleration a0
Example
with respect to the road. Find the angle made by the string with the
vertical.

a0
θ

m
a0
ALTERNATIVE θ

m
Solution:
➔ We shall work from car frame.
➔ Observer A is in car.

T cosθ

+
T
+
θ θ ➔ For observer A, acceleration of mass = 0
a0 ➔ As car is accelerating, so it is non- inertial
ma0 T sinθ
m Frame, So Pseudo force ma0 will be leftwards
A
– mg

T cos θ – mg = m(0)
⇒ T cos θ = mg ….(1)
T sin θ – ma0 = m(0) ….(2)
⇒ T sin θ = ma0
Solution:

T cos θ = mg ….(1)
T sin θ = ma0 ….(2)

Dividing eqn. (2) with (1)

T sin θ ma0
⇒ =
T cos θ mg

a0
⇒ tan θ =
g
a0
θ= tan–1
g
Example A block of mass m is kept on weighing machine, which is placed in lift,
accelerating down with acceleration ‘a’. Find reading of weighing machine.

m
Weighing Machine
ALTERNATIVE

m
Weighing Machine
Solution:

As lift is accelerating, so lift is non- inertial N ma (pseudo)


frame a
Pseudo force will be ma, upward. Consider an +

observer in the lift.For observer, Acceleration of m


block = 0
– Weighing machine

Applying Newton’s 2nd law on block.


mg
N + ma – mg = m(0) = 0
⇒ N = mg – ma
Solution:

⇒ N = mg – ma
∴ Reading = N kg.
g
= mg – ma
kg.
g

Note: If acceleration of lift is g downwards (lift is falling freely), reading of


weighing machine will be zero. (as a = g) : Condition of weightlessness.
A box of mass m is placed on a wedge of mass ‘M’ on a smooth surface.
Example
How much force F is required to be applied on wedge M so that during
motion mass m remains at rest relative to wedge.

A. (M + m)g cosθ
B. (M + m)g sinθ
C. (M + m)g tanθ
D. None of these F

M
θ
F

M
θ
Solution: macosθ

θ

➔ Consider observer on ma
wedge F θ +
➔ As wedge is accelerating, masinθ mgsinθ

wedge is non-inertial M mg θ
frame. So we need to apply mgcosθ
pseudo force.
➔ Pseudo force is ma Applying Newton’s 2nd law on block along incline
leftwards mgsinθ – macosθ = m(0) = 0 ⇒ mgsinθ = macosθ
➔ With the wedge frame,
block is at rest so ⇒ a = gtanθ
acceleration of block along
incline is zero
Solution:
a
Both wedge and block are moving together, so
block + wedge can be considered as system m

Mass of system =M+m


F
Acceleration of System = a M θ
Force on system = (M + m)a = (M + m)g tanθ
Newton’s Laws of Motion
LECTURE 7
Applications of NLM
Applications of NLM
Block of mass ‘M’ is attached with Non–ideal string of mass ‘m’.
Example
Find Tension at (i) point A (ii) point B (midpoint of string).

A. Mg, (M + m/2)g
B. (M + m/2)g, Mg
C. Mg, m/2 g B
D. None of these
A

M
Solution:

Tension at point A :
TA
+ B
Applying Newton’s 2nd law A
M
F = Ma –
Mg
⇒ TA – Mg = M(0) = 0

(acceleration of block = 0)

So, TA = Mg
Solution:
+ B T ➔Let's consider block & string
Tension at point B : A B below point B as the body
M
➔Weight on the body will be
Applying Newton’s 2nd law –
(M +m/2)g, vertically
Mg
⇒ TB – (M + m/2)g = M(0) = 0 downwards
(acceleration of block = 0) ➔As mass of string below
➔point B will be m/2.
⇒ TB = (M + m/2)g
➔So total mass of body will be
Tension at different point of M + m/2.
string will be different
Example Find the Tension at midpoint of uniform rope. (mass of rope = m)

F
M m
Solution:
Lets first, find acceleration of system. To find acceleration,
take block and whole of rope as a system.

Mass of system =M+m


m
Force on system = F M F

Force on system
∴ Acceleration of System =
Mass of system

F
a=
M+m
Solution:

➔To find tension at midpoint, take block midpoint of rope


m
and half of rope as a system M T
➔Tension will be along the string and
a
away from system, i.e. on right side
here. Applying Newton’s 2nd law on this
system.
Uniform rope means mass of rope is
uniformly distributed, So, mass of half of
rope will be m/2. So, mass of this system
= M + m/2
m
T = M+ a
2

m F
T = M+ 2
M+m
Example Find the acceleration of masses m1 & m2

• m1

m2

• m1

m2
Solution:

Apply Newton’s 2nd law for pulley B. •

F = ma
⇒ T + T – T′ = (0) a
[Mass of pulley is zero] T T
T

⇒ 2T – T′ = 0 • m1
T’
⇒ 2T = T′
T’

m2
Solution:

Applying short trick A•


T T T
→ →
∑ T. a =0 ⇒ Ta1 – 2Ta2 = 0 ⇒ Ta1 = 2Ta2 •B
m1 a1

+ ⇒ a1 = 2a2 …(i) 2T
m 1g
m2
Apply Newton’s 2nd law on m2 +
m2g
– m2g – 2T = m2a2 … (ii)
Apply Newton’s 2nd law on m1

T – m1g = m1a1 …
(iii)
Solution:

⇒ a1 = 2a2 …(i)
m2g – 2T = m2a2 … (ii)
T – m1 g = m1a1 … (iii)
T – m1g = 2m1a2
2T – 2m1g = 4m1a2 …(iv)
Solution:

m2g – 2T = m2a2 … (ii) As a1 = 2a2

2T – 2m1g = 4m1a2 ……(4) a = 2(m2 – 2m1)g


1
4m1 + m2
m2g – 2m1g = 4m1a2 + m2a2

(m2 – 2m1)g
a2 =
4m1 + m2
A man of mass M1 starts climbing the rope that goes over ideal pulley & supports
Example a block of mass M2 such that block is always at the same level as man.
M1
Find the ratio of
M2
A. 1
B. 2

C. 1/2
D. None of these

M2
M1
Solution:

Applying Newton’s 2nd law on M2 •

T
T – M2g = M2a + T +
⇒ T = M2g + M2a a M a
1 M2
T
⇒ T = M2(g + a)
⇒ M = T …(1 –
M1 g M2g

2 g+a
)
As man always
Applying Newton’s 2nd law on M1
observes block
T – M1g = M1a at the same level
⇒ T = M1g + M1a as him, both will
⇒ T = M1(g + a) have same
T acceleration
⇒ M1 = g + a …(2)
Solution:

T
⇒ M2 = g + a …(1)

⇒ M1 = gT+ a …(2)

From eq. (1) and (2)

M1 = M 2
M
So, =1
1M
2
Example It is observed, a trolley of mass m, moves with a velocity V=2t as
shown. Find the force acting on the trolley (where, t represents time).
A. 2m
B. 3m
C. 4m
D. 8m
V = 2t
m
Solution:

We know
F = m dv + v dm
dt dt V = 2t
m
As mass trolley m = constant
∴ F = m dv
dt
Magnitude of force
d(2t)
F = m dv = m
dt dt
∴ F = 2m

Example A trolley of mass ‘m’ is going with constant velocity v . Due to rain, water is getting
collected at rate r kg/s in the trolley. Find Force applied on the trolley.


v
Solution:
dm
F = m dv + v
dt dt →
dv = 0 v
As v = constant,
dt
dm
∴ F = v
dt
dm Rate of change of mass of
As =
dt Trolley = r (given)

∴ F =v r
In arrangement shown in figure, the ends A and B of an inextensible string
Example move downwards with uniform speed u. Pulleys A and B are fixed. Find
the speed with which the mass M moves upwards.

A. u tanθ
B. u cosθ
C. u secθ • •

D. u sinθ
θ

u
u
Solution:

➔As the length of the string during


motion . remains constant v
(inextensible string), we can state • •

that velocity components of all the θ


points on a string along length of
string must remains same. m
➔As block is going up with speed v.
Speed of point P will be v upward.
u
And its component along segment u
PR/PQ will be v cos θ

∴ v cos θ =u
u = u sec θ
⇒v =
cosθ
Example If acceleration of ‘B’ is ‘a’ downwards, what will be the
acceleration of A ?
A. a cosθ
B. a cotθ
C. a cosecθ B B

D. a sinθ a
A
θ
Solution:

As given B is accelerating downwards


with an acceleration a. Let acceleration
of A is aA leftwards.

aA sin θ = a cos θ
aA
B
⇒ aA = a cot θ
θ

A θ
Example Figure shows a hemisphere and a supported rod. Hemisphere is moving in right
direction with a uniform velocity v2 and end of rod which is in contact with
ground is moving in left direction with velocity v1. Find the rate at which angle θ
is changing in terms of v1, v2, R and θ.

v1 v2
θ
v1 v2
θ
Solution:
➔ First of all let us find θ In terms of given quantities.
➔ Let x is distance between points A and B
➔ Rate of change of x will be equal to v1 + v2
➔ x will depend on both v1 and v2
dx = v1 + v 2
dt
d R cosecθ R
= v1 + v2 v1 θ v2
dt x
d cosecθ
R = v1 + v 2
dt
Applying chain law

d cosec θ dθ
R = v1 + v2
dθ dt
Solution:

R D cosec θ dθ = v1 + v 2
dθ dt
R
dθ = v1 + v 2 θ v2
R (–cosec θ. cot θ) v1
dt x

dθ –(v1 + v2) (negative sign signifies q will decrease with time)


=
dt R (cosec θ. cot θ)
Example In the system shown in figure , if wedge A is displaced towards right at
acceleration 1 m/s2. Find acceleration of block B w.r.t. ground ?

A. √3 m/s2
B. √5 m/s2
C. √7 m/s2
D. 2 m/s2

A aA = 1m/s2
600
XA
XA

XA

A aA = 1m/s2
600
➔As wedge A is displaced by xA right
wards, so WITH RESPECT TO wedge
A point O, P will appear to be
displaced by xA left wards
➔Points O, P will appear to be
displaced by xA away from centre of
their respective segments so
contribution to elongation will be +
xA for each point.
➔Let block B is displaced
upwards(along the incline) with xBA
(relative to wedge A), so point Q will
displaced upwards with Xba
➔As point Q is displaced towards
centre of its segment so contribution
to elongation will be – xBA.
Constrained relationship : →
aBA
+ xA + xA –xBA = 0
600 →
xBA = 2xA aAG
vBA = 2vA
aBA = 2aA = 2´ 1 = 2 m/s2

Acceleration of B with respect to Ground


aBG = aBA + aAG

aBG = √ (2)2 + (1)2 + 2 (2) (1) cos 60


= √7 m/s2
In the system shown in figure, the block A is pulled towards right at a
Example
constant speed of 1 m/s. Find speed of block B w.r.t. ground.

A. 8 m/s


B. √37 m/s
C. √23 m/s


D. 6 m/s


A B


VA =


1m/s
X


X
X


B

X
A


X

X X
Solution:

Velocity of B with respect to A

➔As wedge A is displaced by x1 right wards, so with the


respect to wedge A point N, O, P, Q, R, S will appear
to displaced by x1 left wards (relative to wedge A)
➔Points N, O, P, Q, R, S will appear to displaced by x1
away from centre of their respective segments so
contribution to elongation will be + x1 for each point
➔As point M is displaced towards centre of its segment
so contribution to elongation will be – x2.
➔Let block B is displaced upwards with x2. So point M
will displaced upwards with x2
Solution:

Velocity of B with respect to A

x1 N
x
x1 • •2
O
x1 P •
M
x1 •
B
Q x1
x1 R •

x1 • VA = 1m/s
A
S
Solution:

Velocity of B with respect to A Velocity of B with respect to Ground

→ VBG = VBA + VAG


VBA ∴ |VBG| = √(6)2 + (1)2
x1 N
x
x1 • •2 900
O → =
x1 P •
M VAG √37 m/s
x1 •
B
Q x1
x1 R •

x1 • VA = 1m/s
A
S

Constrained relationship :
+ 6 x1 – x2 = 0 ⇒ 6 x1 = x2⇒ 6 v1 = v2
∴ VBA = 6 VA = 6 × 1 = 6 m/s
Newton’s Laws of Motion
LECTURE 8
Spring Force
Spring Force
fixed
Spring Force
When no force acts on a spring, it is in relaxed condition i.e. neither
compressed nor elongated.
When it is pushed by a force F, it is
compressed and displacement x of its free
end is called compression.
When the spring is pulled by a force F,
it is elongated and displacement x of
its free end is called elongation.
Relaxed spring x=0 Equilibrium / Mean position

Compressed spring Compression

Force by spring on the


wall

Force by wall Force by hand Force by


on spring on spring spring on
hand
Elongation
Elongated spring

Force by wall on
spring

Force by spring on Force by spring on Force by hand on


wall hand spring
Hooke’s Law
Hooke’s Law
Hooke’s Law
Relaxed spring x = 0 Equilibrium / mean position
How spring force F varies with
deformation in length x of the
spring is also shown in the Compression
Compressed spring
following figure.
Elongation

Elongated spring

Spring force

Compression Extension
Hooke’s Law
The force F varies linearly with x and acts in a direction opposite to x.
Therefore, it is expressed by the following equation.
F = −kx

Here, the minus (−) sign represents the fact that force F is opposite to x.
The constant of proportionality k is known as spring constant
Spring Constant (Stiffness)

The constant of proportionality k is known as spring constant.


The slope modulus of the graph equals to the spring constant.

SI unit of spring constant k is newton per meter or (N/m).

Dimensions of k are MT−2


Example A mass M is suspended as shown in figure. The system is in equilibrium.
Assume pulleys to be massless. k is the force constant of the spring.
The extension produced in the spring is given by

A. 4Mg/k
B. Mg/k
C. 2Mg/k
D. 3Mg/k

M
Example A mass M is suspended as shown in Figure. The system is in
equilibrium. Assume pulleys to be massless. K is the force
constant of the spring. The extension produced in the spring is
given by
A. 4Mg/K
B. Mg/K
C. 2Mg/K
D. 3Mg/K
The system is in equilibrium. Pulleys are massless. K is the force constant
Example of the spring. Find the net tension force acting on the lower support.
A. Mg
B. 2Mg
C. 3Mg
D. 4Mg

M
Example The system is in equilibrium. Pulleys are massless. K is the
force constant of the spring. Find the net tension force acting
on the lower support.
A. Mg
B. 2Mg
C. 3Mg
D. 4Mg
Example The system is in equilibrium. K is the force constant of the spring. If each
of the pulley A and B has mass M, Find the net tension force acting on the
lower support.
A. 2Mg
B. 6Mg
C. 3Mg
D. 4Mg

M
Example The system is in equilibrium. K is the force constant of the
spring. If each of the pulley A and B has mass M, Find the net
tension force acting on the lower support.
On applying a force F the point ‘P’ is displaced vertically down by y from
Example equilibrium position Find the force F in terms of the force constant k of
the spring & displacement y, for the cases (A) and (B)
On applying a force F the point ‘P’ is displaced vertically down by y from
Example equilibrium position Find the force F in terms of the force constant k of
the spring & displacement y, for the cases (A) and (B)

Solution:
Case. (A)
At point P : F =T …..(ⅰ)
And for the equilibrium of the pulley , 2T = Fs …..(ⅱ)
But as due to shift of point P by y , the spring stretches by (y/2) , so
Fs = k(y/2) ….(ⅲ)
So Substituting Fs from Eqn. (ⅲ) in Eqn. (ⅱ)
And then T from Eqn. (ⅱ) in Eqn. (ⅰ), we get F = (k/4)y ...(A)

Case(B) as tension in massless string and spring will be same, T = F’s …(ⅰ)
For Pulley : F = 2F’s
...(ⅱ)
Now if mass M shifts by y the spring will stretch by 2y (as string is inextensible)
F’s = k(2y)
…(ⅲ)
So, Substituting F’ from Eqn. (ⅱ) in Eqn. (ⅲ), F=(4k)y ….(B)
Example One end of a massless spring of natural length 3R/4 is attached to the lowest
point O of the wire track. A small ring of mass m is held stationary of point P
such that the spring makes an angle of 60° with the vertical. k = (mg/R).
At the instant when the ring is released :-
Determine the tangential acceleration of the ring &
the normal reaction between ring and track.

g
Solution:
1. The free body diagram of the ring is shown in figure. The forces
acting on the ring are:
a. The weight mg acting vertically downwards
b. Normal force N by the wire track.
Normal force on the ring could be either radially outwards or →
radially inwards depending on whether the ring presses against
the inner surface or outer surface of the track. To ascertain
whether normal force is inwards or outwards assume that, to

begin with, it is inwards, then from ∑F = ma find the Value of
normal force, if it is positive it is inwards if it is negative, it is
outwards.
a. Force of the spring kx. In the given physical situation, the
spring is extended, it will pull the ring. So the spring force kx is
along the spring towards O.
Solution:
2. Length of the spring in the position shown = R.(CP = CO = R; ∠COP =
∠OPC = 60°; ∠COP is equilateral)
Change in length of the spring
=

Now from Ft = mat,

cos 30° + mg cos 30° = mat ⇒ at =

Now consider radial direction N + kx cos 60° = mg cos 60°

N = mg cos 60° - kx cos 60 ° =


String Cut Problems


m
String Cut Problems
Tension Spring Force

Instantly Vanishes/Changes Stays

m
Example 2 blocks are connected by a spring. The combination is suspended, at rest,
from a string attached to the ceiling, as shown in the figure. The string
breaks suddenly. Immediately after the string breaks, what is the initial
downward acceleration of the upper block of mass 2m?

2m

m
Example Two blocks are connected by a spring. The combination is suspended,
at rest, from a string attached to the ceiling, as shown in the figure.
The string breaks suddenly. Immediately after the string breaks, what
is the initial downward acceleration of the upper block of mass 2m?
Solution:
Step-I: Discuss the problem before cutting the
string: From force diagram of lower block,Kx0
= mg. From force diagram of upper block: T = T Kx0
2mg + Kx0

Step-II: Discuss the problem after cutting the


A B
string, 2mg + Kx0 = 2ma or 2mg + mg = 2ma

or 3mg = 2ma Kx0

2mg mg
Example The system of two weights with masses m1 & m2 are connected with
weightless spring as shown the system is resting on the support ‘S’. Find
the acceleration of each of the weights just after the support ‘S’ is quickly
removed.

m1

m2

support
Solution:
Force of spring does not change instantaneously so find spring force at initial
instant Initially m1g = kx when support is removed spring force does not change

For m1 : m1g - kx = m1a1 or a1 = 0


M1g M1g

For m2 : m2g - kx = m2a2 or a2 =


M1 M1

kx kx
m1 M2g
M2g kx
kx
m2 M2 M2

N=0
FBD initially
FBD when support is
support
removed
Example If string connecting blocks m2 & m3 is cut at point A. Find the
accelerations of masses m1, m2, & m3 just after the string is cut!

A
m2

m3

m1
Example Three blocks are arranged with pulley and spring as shown in the
figure. If string connecting blocks m2 and m3 is cut at point A. Find
the accelerations of masses m1, m2, and m3 just after the string is
cut at Point A.
Solution:
Let us analyse the system at equilibrium. The forces acting on the


blocks are shown in figure.
Just after cutting the string at A, the tensions T and T’ will be
A
zero. But the spring force will remain unchanged just after cutting. m2
Figure shows the forces acting on the blocks just after cutting the m3
string.

m1

The forces acting on m1 just before and just after cutting the string
is same and just before cutting the string, equilibrium, hence the
acceleration of m1 will be zero just after cutting the string.
Solution:
At equilibrium Fs = m1g
As Fs does not change just before and just after cutting the string,
acceleration of m2

From free-body diagram of block m3 just after cutting the string, only
m3g will act and

tensions in the string will disappear. Hence acceleration of m3 a3 =

Finally, acceleration of m1 = 0

Acceleration of m2 = and acceleration of m3 = g


Example The block m2 hangs from a fixed smooth by an inextensible string that is
fitted with a light spring of stiffness k as shown. Find the acceleration of
the bodies just after the string S is cut.

m1


m2
Example Two blocks of masses m1 and m2 are in equilibrium. The block m2 hangs
from a fixed smooth by an inextensible string that is fitted with a light
spring of stiffness k as shown in the Fig. Neglecting friction and mass of
the string, find the acceleration of the bodies just after the string S is cut.

Solution:
FBD: Let the spring forces be F = kx just after cutting the spring. Hence,
at that instant the forces acting on m1 are T = kx → m1g 4↓ and N↑: on m2
the forces arc m2g ↓ and T↑.

Force equation: Initially, all the S kx T


particles are stationary; m1


T
T
m2

m2g
Solution:
Hence a1 = a2 = 0. Applying Newton’s 2nd law
For m1: ΣF = T’ - kx = 0 …(ⅰ)

For m2: ΣF = T’ - m2g = 0 …(ⅱ)

For spring : ΣF = T - kx = 0 …(ⅲ)

Solving eq.(ⅰ), (ⅱ) and (ⅲ) we have


T’ = T = kx = m2g as shown in Fig.
Just after the string S is cut, tension T’ vanishes immediately, but the spring cannot
regain its shape and size instantaneously. Therefore, the spring remains as it is. Then,
just after cutting the string, the net force acting on m1 is equal to kx whereas the net
force acting on m2 is equal to T-m2g. Hence, the acceleration of m1 and m2 just after
cutting the string is given by
ΣF = m1a1 = kx = m2g,
ΣF = m2a2 = T - m2g = 0

This gives and a2 = 0


Composite Springs
Springs in Parallel
keq F
Springs in Parallel
This situation is shown in figure If the
force F pulls the mass m by y, the
stretch in each spring will be y,
i.e., y1 = y2 = y

Now as for a spring F = ky and as k’s are not equal so F1 ≠ F2 but for
equilibrium

F = F1 + F2 i.e., ky = k1y1 + k2y2 [as F = ky]

which in the light of Eq. (i) reduces to k = k1 + k2 + ....

This is like capacitors in parallel or resistance in series.

keq = k1 + k2 + ....
Springs in Series
Springs in Series

keq F
Springs in Series
This situation is shown in figure

As springs are mass less, force in these must be


same, i.e.,

F1 = F 2 = F (i)

Now as F = ky and k’s are not equal, stretches will not be equal,
i.e., y ≠ y2

But, y = y1 + y2 or [as for F = ky, y = (F/k) ]

which in the light of Eq. (i) reduces to


4 identical springs are connected as shown each spring has a spring
Example constant of 10 N/m. Find the net extension of the spring system on
attaching a 25 kg mass

A. 5m
B. 10m
C. 15m
D. 25m

25 kg
Newton’s Laws of
Motion

DPP 1
Basic Problems
A boy sitting on the topmost berth in the compartment of a train which is
just going to stop on a railway station, drops an apple aiming at the open hand
of his brother sitting vertically below his hands at a distance of about 2 meter.
The apple will fall
A. Precisely on the hand of his brother
B. Slightly away from the hand of his brother in the direction of motion of the train
C. Slightly away from the hand of his brother in the direction opposite to the direction of
motion of the train
D. None of the above
A person sitting in an open car moving at constant velocity throws a
ball vertically up into air. The ball falls

A. Outside the car


B. In the car ahead of the person
C. In the car to the side of the person
D. Exactly in the hand which threw it up

v
A mass of 1 kg is suspended by a string A. Another string C is connected
to its lower end (see figure). If the string C is stretched slowly, then
A. The portion AB of the string will break
The portion BC of the string will break A
B.
C. None of the strings will break
D. None of the above
B
1kg

C
A machine gun is mounted on a 2000 kg car on a horizontal frictionless
surface. At some instant the gun fires bullets of mass 10 gm with a velocity of
500 m/sec with respect to the car. The number of bullets fired per second is
10. The average thrust on the system is
A. 550 N
B. 50 N
C. 250 N
D. 250 dyne
A machine gun fires a bullet of mass 40 g with a velocity 1200 ms-1. The man
holding it can exert a maximum force of 144 N on the gun. How many bullets can
he fire per second at the most?
A. One
B. Four
C. Two
D. Three
A body of mass 2 kg has an initial velocity of 3 m/s along horizontal
direction and it is subjected to a force of 4 N in vertical direction.. The
distance of the body from O after 4 seconds will be
A. 12 m
B. 20 m 3 m/s
C. 8m 2 kg
D. 48 m
The motion of a particle of mass m is given by x = 0 for t < 0 s,
x (t) = A sin 4πt for 0 < t < (1/4)s (A> 0), and x = 0 for t > (1/4) s.
Which of the following statements are TRUE ?
Multi-Correct
A. The force at t = (⅛ ) s on the particle is —16π2 Am

B. The force at t = (⅛) on the particle is 16π2Am

C. The particle is not acted upon by any force

D. The particle is not acted upon by a constant force


The motion of a particle of mass m is given by x = 0 for t < 0 s,
x (t) = A sin 4πt for 0 < t < (1/4)s (A> 0), and x = 0 for t > (1/4) s. Which of the following
statements are true?
Multi-Correct

A. The force at t = (⅛ ) s on the particle is —16π2 Am

B. The force at t = (⅛) on the particle is 16π2Am

C. The particle is not acted upon by any force

D. The particle is not acted upon by a constant force


Newton’s Laws of
Motion

DPP 2
Free Body Diagram
N

mg
A particle of mass m = 5 kg, at rest at x=0 at t= 0. It is acted upon by 2
forces F1 and F2 It is given that & F2 is unknown. The particle
experiences a constant acceleration a , in the direction as shown.
What third force F3 is required to make the acceleration of the particle zero?

530
5kg
x
3 forces starts acting simultaneously on a particle moving with velocity
v These forces are represented in magnitude & direction by the 3 sides of a
triangle ABC (as shown). The particle will now move with velocity
A. v remains unchanged
C
B. Less than v

C. Greater than v
D. v in the direction of the largest force BC

A B
A helicopter is moving to the right at a constant horizontal velocity. It
→ → →
experiences three forces Fgravitational, Fdrag and force on it caused by rotor
Frotor. Which of the following diagrams can be correct free body diagram
representing forces on the helicopter?

Direction
of motion
Consider the system as shown in the figure. The pulley and the string are
light and all the surfaces are frictionless. The tension in the string is
(g = 10 m/s2).
A. 0N
B. 1N
C. 2N 1kg

D. 5N

1kg
In the shown mass pulley system, pulleys and string are massless. The
one end of the string is pulled by the force F = 2mg. The acceleration of
the block will be
A. g/2
B. 0
C. g
D. 3g

F = 2mg

m
In the figure shown, a balloon is pressed against a wall. It is in equilibrium & maximum
→ → →
compressed state. F1 = force of balloon on hand man, F2 = force of balloon on wall, F3 =

friction, F4 = weight of balloon. Which of the following statements are correct? Multi-Correct

A.
B.
C.
D.
In the figure shown, a balloon is pressed against a wall. It is in equilibrium
→ →
and maximum compressed state. F1 = force of balloon on hand man, F2 =
→ →
force of balloon on wall, F3 = friction, F4 = weight of balloon. Which of the
following statements are correct? Multi-Correct

A.

B.
C.
D.
Blocks A and C start from rest and move to the right with acceleration
aA = 12t ms-2 and aC = 3 ms-2. Here t is in seconds. The time when block B
again comes to rest is ?
A. 2 sec
A C
B. 1 sec
C. 3/2 sec
D. 1/2 sec

B
ALTERNATIVE 12t ms-2 3 ms-2

A C

B
Newton’s Laws of
Motion

DPP 3
String & Pulley Problems

m2

θ
A weight W is supported by two strings inclined at 600 and 300 to the
vertical. The tensions in the strings are T1 and T2 as shown. If these
tensions are to be determined in terms of W using a triangle of forces,
which of these triangles should you draw? (block is in equilibrium).
T2
A. B.
T1 T1
W W T2
T1 600
T2 300

T1 T2
C. D.

W W T1
T2

W
Which of the following sets of concurrent forces may be in equilibrium?

A. F1 =3N, F2=5N, F3=9N

B. F1=3N, F2=5N, F3=1N

C. F1=3N, F2=5N, F3=15N


D. F1= 3N, F2= 5N, F3= 6N
A uniform sphere of weight w and radius 3m is being held by a string of
length 5m attached to a frictionless wall as shown in the figure.
The tension in the string will be:
A. 5w/4
B. 15w/4
C. 15w/16
D. None of the above
A 50kg person stands on a 25kg platform. He pulls on the rope via frictionless
pulleys as shown in the figure. The platform moves upwards at a steady rate
if the force with which the person pulls the rope is ?
A. 500 N
B. 250 N
C. 125 N
D. None of these
50 kg

25 kg
ALTERNATIVE

50 kg

25 kg
Find the tension in the string AB loaded with weight W at the middle,
when AB is horizontal:
A. zero
A B
B. W
C. W/2
D. Infinity W
2 smooth spheres each of radius 5cm and weight W rest one on the
other as shown. The reactions between the spheres and the walls 1 & 2
are:

A. W/4 and 3W/4


2
1
B. W/4 and W/4
C. 3W/4 and 3W/4
D. W and W

16 cm
In the figure, at the free end of the light string, a force F is applied to
keep the suspended mass of 18 kg at rest. Then the force exerted by the
ceiling on the system (assume that the string segments are vertical and
the pulleys are light and smooth) is: (g = 10 m/s2)
A. 60 N
B. 120 N
C. 180 N
D. 240 N

18kg
A cylinder of mass M and radius R is resting on two corner edges A and B
as shown in figure. The normal reaction at the edges A and B are:
(Neglect friction) Multi-Correct
A.

B.
C.

D. 600 300

A
B
A cylinder of mass M and radius R is resting on two corner edges A and B
as shown in figure. The normal reaction at the edges A and B are:
(Neglect friction) Multi-Correct
A.

B.
C.

D. 600 300

A
B
Newton’s Laws of
Motion

DPP 4
Constrained Motion - 1

A

B 3 m/s2
A rod AB is shown in figure. End A of the rod is fixed on the ground. Block is
moving with velocity 2 ms-1 towards right. The velocity of end B of rod at the
instant shown
in figure is ?
B v = 2 m/s
A. 3 ms-1
B. 2 ms-1
C. 2 3 ms-1
D. 4 ms-1
A 300
A rod AB is placed against a block which is moving towards right with a
speed of 1 m/s. If at an instant when the rod makes an angle 600 with the
horizontal & end A is sliding left with a speed of 1 m/s. Then the speed of
the point B of the rod is ?
A. 1 m/s B
B. 3 /2 m/s 1 m/s
C. 1/2 m/s
D. 7 3 m/s
1 m/s
600
A
B

1 m/s

1 m/s
600
A
B B

1 m/s

1 m/s 2 m/s
600 600
A A
A car is moving on a plane inclined at 30° to the horizontal with an
acceleration of 10m/s2 parallel to the plane upwards. A bob is suspended
by a string from the roof. The angle in degrees which the string makes with
vertical is: (assume that bob doesn't move w.r.t. car) [g = 10 m/s2]

A. 20°
B. 30°
C. 45°
D. 60°

30°
A cylinder rests in a car as shown. The car moving with an horizontal acceleration 'a'
The magnitude of normal reactions exerted by sides AB and BC of carriage on the
cylinder be NAB and NBC respectively. Neglect friction everywhere then as the
magnitude of acceleration 'a' is increased, pick the CORRECT statement.

A. NAB increases and NBC decreases


B. Both NAB and NBC increases
C
A
C. NAB remains constant and NBC increases
D. NAB increases and NBC remains constant
a 30°
B
C
A

a
30°
B
System is shown in figure and man is pulling the rope from both sides
with constant speed ‘u’ .Then velocity of the block will be?
A. 3u/4 u u
B. 3u/2
C. u/4
D. u/2

M
u u ALTERNATIVE

M
A rod can freely rotate in vertical plane about the hinge at its bottom.
Two strings tie the top of the rod with blocks A and B as shown in the
figure. At the instant shown, the rod is vertical and the speed of block A
is u m/s downwards. Find the speed of block B.
A. √3u
30° 60°
B. u
u
C.
2√2

2u A B
D.
√ 3 u
A lift is moving upwards with a constant speed of 5 m/s. The speed of one
of the pulleys is 5 m/s as shown. Then the speed of second pulley is :

A. 5 m/s
B. Zero
C. 7.5 m/s 5 m/s 5 m/s

D. 10 m/s
In the figure shown neglecting friction and mass of pulleys, what is the
acceleration of mass B ?
g
A. 3
5g
B.
2 m
A

C. g

D. 2g m B
5
Figure shows two blocks A and B connected to an ideal pulleys sting
system. In this system when bodies are released then : (neglect friction
and take g = 10 m/s2) Multi-Correct
B
A. Acceleration of block A is 1 m/s
2
10kg
B. Acceleration of block A is 2 m/s 2

C. Tension in string connected to block B is 40N


D. Tension in string connected to block B is 80N

A 40Kg
a
Figure shows two blocks A and B connected to an ideal pulleys sting
system. In this system when bodies are released then : (neglect friction
and take g = 10 m/s2) Multi-Correct B
A. Acceleration of block A is 1 m/s 2

B. Acceleration of block A is 2 m/s2 10kg

C. Tension in string connected to block B is 40N


D. Tension in string connected to block B is 80N

A 40Kg
a
Newton’s Laws of
Motion

DPP 5
A fixed U-shaped smooth wire has a semi-circular bending between A and
B as shown in Figure. A bead of mass m moving with uniform speed v
through the wire enters the semicircular bend at A and leaves at B. The
average force exerted by the bead on the part AB of the wire is ?
[g = 10 m/s2]
A. 0 v
4mv2
B.
πd
A

C. 2mv
2

πd d

D. None of these B
3 blocks A, B, and C are suspended as shown. Mass of each of blocks A & B is
m. If the system is in equilibrium, and mass of C is M, then ?

A. M > 2m
B. M = 2m
C. M < 2m
D. None of these

A m C M B m
In the figure shown, find out the value of v [Assume string to be tight]

A. 4.25 m/s 1 m/s

B. 3.25 m/s
C. 3 m/s
D. None of the above

37° 30°
v=? B
A
ALTERNATIVE
1 m/s

37° 30°
A v=? B

37° 30°
v=?
A B

1 m/s 1 m/s
Two small rings O and O’ are put on two vertical stationary rods AB and A’ B’,
respectively. One end of an inextensible thread is tied at point A’. The thread
passes through ring O’ and its other end is tied to ring O. Assuming that ring
O’ moves downwards at a constant velocity v1, then Find velocity v2 of the
ring O, when ∠AOO’ = α, is ?
A A/
A.

B.
v2 o/
C. α
o
D. None of these v1
The velocity of point A on the rod is 2 ms-1 (leftwards) at an instant. The
velocity of the point B on the rod at this instant is ?

A.
B
B. 1 ms-1

C.

D.

VA= 2ms-1 600


A
One monkey of mass m can climb strings of pulley arrangements shown
in figure. Find magnitude of acceleration (in m/s2) of the monkey with
respect to rope so that block of mass 5m remains stationary.

A. 90 m/sec2
B. 100 m/sec2
C. 110 m/sec2
D. 120 m/sec2

5m
A force produces an acceleration of 4 ms-2 in a body of mass m1 and the
same force produces an acceleration of 6 ms-2 in another body of mass m2.
If the same force is applied to (m1 + m2), then the acceleration will be:

A. 10 ms-2 F
m1
B. 2 ms-2
C. 2.4 ms-2
D. 5.4 ms-2 F
m2

m1 F
m2
2 blocks ‘A’ & ‘B’ each of mass ‘m’ are placed on a smooth surface. 2
horizontal forces F & 2F are applied as shown in figure. The block A doesn’t
slide on block B. the normal reaction acting between the two blocks is:
A. F
B. F/2
C. F / √3
D. 3F
A B

F 2F
m m
30o
A light string is wrapped around a cylindrical log of wood which is placed
on a horizontal surface with its axis vertical and it is pulled with a
constant force F as shown in the figure. (Friction is absent everywhere) MULTI-CORRECT
A. tension T in the string increases with increase in 𝛉
B. tension T in the string decreases with increase in 𝛉
C. tension T > F if 𝛉 > π/3
D. tension T > F if 𝛉 > π/4

θ
θ F
A light string is wrapped around a cylindrical log of wood which is placed
on a horizontal surface with its axis vertical and it is pulled with a
constant force F as shown in the figure. (Friction is absent everywhere) MULTI-CORRECT
A. tension T in the string increases with increase in 𝛉
B. tension T in the string decreases with increase in 𝛉
C. tension T > F if 𝛉 > π/3
D. tension T > F if 𝛉 > π/4

θ
θ F
A fireman wants to slide down a rope. The rope can bear a tension of
3/4th of the weight of the man. With what minimum acceleration should
the fireman slide down:
A. g/3
B. g/6
C. g/4
D. g/2
A system is shown in the figure. A man standing on the block is pulling
the rope. Velocity of the point of string in contact with the hand of the
man is 2 m/s downwards. The velocity of the block will be:
[assume that the block does not rotate]
A. 3 m/s
B. 2 m/s
C. 1/2 m/s
D. 1 m/s
A wedge of mass 2kg resting on a frictionless floor. A block of mass 1 kg is kept on
the wedge and the wedge is given an acceleration of 5 m/s2 towards right. Then
A. block will remain stationary w.r.t. wedge
B. the block will have an acceleration of 1 m/sec2 w.r.t. the wedge
C. normal reaction on the block is 11 N
D. net force acting on the wedge is 2 N

5 m/s2

2 kg
370
The masses of the block are m1 = 20 kg & m2 = 30 kg.
The accelerations of masses m1 & m2 will be if F = 180 N

A. a1 = 9 m/s2, a2 = 0
B. a1 = 9 m/s2, a2 = 9 m/s2
C. a1 = 0, a2 = 9 m/s2 m1
D. a1 = 0, a2 = 4.5 m/s2

m2
F
System shown in figure is in equilibrium and at rest. The spring and string are
massless Now the string is cut. The acceleration of mass 2m & m just after the
string is cut will be : -2006
A. g/2 upwards, g downwards
B. g upwards, g/2 downwards
C. g upwards, 2g downwards
D. 2g upwards, g downwards

2m

m
A pendulum of mass m hangs from a support fixed to a trolley. The
direction of the string when the trolley rolls up a plane of inclination α with
acceleration a0 is (String and bob remain fixed with respect to trolley):

α
Newton’s Laws of
Motion

DPP 6
The tension in the spring is

A. Zero
B. 2.5 N
C. 5 N
D. 10 N
A uniform rope of mass M is placed on a smooth fixed wedge as shown.
Both ends of the rope are at same horizontal level. The rope is initially
released from rest, then the magnitude of initial acceleration of rope is?

A. Zero

B. M(cos𝜶 - cos𝞫)g
C. M(tan𝜶 - tan𝞫)g
D. None of these

𝜶 𝜷
A rope over a pulley, as shown. One end of the rope is held by Student A of
mass 70 kg, who is at rest on the floor. The opposite end of the rope is held
by Student B of mass 60 kg, who is suspended at rest above the floor. The
minimum acceleration a0 with which the student B should climb up the rope
to lift the student A upward off the floor.

A. 1/3 m/s2
B. 2/3
m/sm/s
C. 4/3
2 2

D. 5/3 m/s2 Student A Student B


70kg 60kg
An ideal spring is compressed and placed horizontally between a vertical fixed wall & a
block free to slide over a smooth horizontal table to as shown in the figure. The system is
released from rest. The graph which represents the relation between the magnitude of
acceleration a of the block and the distance x travelled by it (as long as the spring is
compressed) is
compressed

A.
a
x

B.
a
x

C. a
x

D.
a
x
fixed
2 blocks of mass M and m are used to compress 2 different massless springs as
shown. The left spring is compressed by 3 cm, while the right spring is
compressed by an unknown amount. The system is at rest, and all surfaces fixed
and smooth. Which of the following statements are true? Multi-Correct

A. The force exerted on block of mass m by the right spring is 6N to the left
B. The force exerted on mass m by the right spring is impossible to determine.
C. The net force on block of mass m is zero
D. The normal force exerted by block of mass M on block of mass m is 6N

K = 2N/cm
K=?
M
m
Two blocks of mass M and m are used to compress two different massless
springs as shown. The left spring is compressed by 3 cm, while the right spring is
compressed by an unknown amount. The system is at rest, and all surfaces fixed
and smooth. Which of the following statements are true? Multi-Correct

A. The force exerted on block of mass m by the right spring is 6N to the left
B. The force exerted on mass m by the right spring is impossible to determine.
C. The net force on block of mass m is zero
D. The normal force exerted by block of mass M on block of mass m is 6N

K = 2N/cm
K=?
M
m
Two blocks A and B of equal mass m are connected as shown in figure.
The wedge is fixed on horizontal surface. Friction is absent everywhere.
When the system is released from rest
Multi-Correct

A. Tension in string is mg/2


B. Tension in string is mg/4
C. Acceleration of A is g/2
D. Acceleration of A is ¾ g
Two blocks A and B of equal mass m are connected through a massless
string and arranged as shown in figure. The wedge is fixed on horizontal
surface. Friction is absent everywhere. When the system is released
from rest Multi-Correct

A. Tension in string is mg/2


B. Tension in string is mg/4
C. Acceleration of A is g/2
D. Acceleration of A is ¾ g
Figure shows a 5 kg ladder hanging from a string that is connected with a ceiling & is
having a spring balance connected in between. A boy of mass 25 kg is climbing up the
ladder at acceleration 1 m/s2. Assuming the spring balance and the string to be
massless and the spring to show a constant reading, the reading of the spring balance
is : Take (g =10 m/s2)

A. 30 kg
B. 32.5 kg
C. 37.5 kg
D. 35 kg
Friction
LECTURE 1
Magnitude of Friction
Friction
Friction – Introduction and Cause
Do you find it difficult to move heavier objects?
Friction
Friction is a property related to two surfaces in contact, which opposes the
relative motion (or tendency to slip) between them.
The force responsible is called frictional force

Friction is due to
Interlocking
Cold welding

In case of fluids we call it viscosity


Interlocking

➔ Every surface has irregularities (projections and depressions).

Interlocking

➔ The interlocking between these irregularities gives rise to friction.


➔ When the surface is smooth the interlocking decreases, leading to decreased friction.
Cold Welding

Cold welding
Bonds are formed at contacts
Because of high pressure.

➔ When 2 surfaces are in contact only few points touch each other
➔ This leads to high pressure at these points forming bonds.
➔ This phenomenon is called as cold welding
➔ Bonds formed at contact points due to high pressure causes friction
➔ This frictional force is independent of area of contact
Magnitude of Friction
Types of Friction

Static Friction Kinetic Friction


Types of Friction
Kinetic Friction
When there is Relative Motion Friction always acts so as to oppose the relative motion

v
M

Rough Floor

Static Friction
When there is no Relative Motion Friction always acts so as to oppose the tendency of
relative motion

Rough Floor
Magnitude of Friction
Kinetic Friction

fk= μkN

μk = Coefficient of Kinetic Friction


Magnitude of Friction
Static Friction fs varies based upon scenario to scenario
fs ≤ μsN
μs = Coefficient of Static Friction

μk ≤ μs
Analysis of Friction Shift from Static to Kinetic
f a
static kinetic static kinetic

μsN
( μ s − μk ) N
μkN
m

t t
μsN μsN
α N α

F= αt
f μ
Kinetic Friction
Motion

Pushing force

Friction
Kinetic Friction
When there is Relative Motion

m v

2v
m1
μ
m2 v
Kinetic Friction
When there is Relative Motion

θ
Kinetic Friction
When there is Relative Motion

Box is
Slipping
v

. .

Whenever a body is slipping over some other surface then it implies that friction is kinetic.
Static Friction
Static Friction
When there is no Relative Motion but there is some tendency

rest rest
F F
m m
μ
Static Friction
When there is no Relative Motion but there is some tendency F > mg sinθ

μ μ
θ θ
Static Friction
When there is no Relative Motion but there is some tendency

No Slipping
m1 F
μ1 No Slipping F
m2 m1 μ1

m1 No Slipping
μ1
m2 F
Static Friction
When there is no Relative Motion but there is some tendency

No
Slipping
a
m
a

. .

The condition of no slipping implies that the friction is static


Direction Of Friction
Direction Of Friction
When there is Relative Motion – Kinetic Friction
Friction always acts so as to oppose the relative motion
Motion
Pushing force

Friction

When there is no Relative Motion – Static Friction


Friction always acts so as to oppose the tendency of relative motion
Direction Of Friction
When there is Relative Motion – Kinetic Friction

rough A vA If vA = vB then no friction as no relative motion


B vB as well as no tendency
smooth
Direction Of Friction
vA > vB rough vA
A

vB
B

smooth
Direction Of Friction
vA< vB vA
rough
A

vB
B

smooth
Direction Of Friction
When there is Relative Motion – Kinetic Friction
rough A vA
If vA = vB then no friction as no relative motion
B vB as well as no tendency
smooth

vA > v B vA < v B
vA vA
f A A
f
f
f
B B vB
vB
Direction Of Friction
When there is no Relative Motion – Static Friction
If FA = FB then no friction as no tendency of
relative motion (masses same)

rough FA
A
No slipping
B FB
smooth
Direction Of Friction
FA > FB (masses same)
rough
A FA
No slipping
B FB
smooth
Direction Of Friction
FA < FB (masses same)
rough
FA
A
No slipping FB
B
smooth
Direction Of Friction
When there is no Relative Motion – Static Friction

rough A FA If FA = FB then no friction as no tendency of


No slipping B FB relative motion ( masses same )
smooth
Direction Of Friction
When there is no Relative Motion – Static Friction

rough A FA If FA = FB then no friction as no tendency of


No slipping B FB relative motion (masses same)
smooth

F A > FB F A < FB

A FA A FA
f f
f f
B FB B FB
Magnitude of Friction – Kinetic Friction
REVISE

m v

μ
Magnitude of Friction – Kinetic Friction

Magnitude f = μkN
k
Direction Opposite to Relative Velocity

μ
θ
v

μ
θ
Magnitude of Friction – Kinetic Friction
N Magnitude f = μkN
k
Direction Opposite to Relative Velocity
f m v N – mg = 0 ➔ N = mg ➔ fk = μmg
k
μ
N
mg

N
μ
v f θ
k mg cos θ
μ
θ

mg cos θ
N – mg cos θ = 0 ➔ N = mg cos θ ➔ fk = μ (mg cos θ)
Magnitude of Friction – Kinetic Friction
2v
Magnitude fk= μkN
m1
μ1 v Direction Opposite to Relative Velocity
m2
μ2
Magnitude of Friction – Kinetic Friction
2v
Magnitude fk= μkN
m1
μ1 v Direction Opposite to Relative Velocity
m2
μ2
N2
N1
f1
μ1
m2
f1 m1 f2
μ2
μ1 N1

m2 g
m1g

N1 – m1g = 0 ➔ N1 = m1g ➔ fk1 = μ1m1g N2 - N1 – m2g = 0 ➔ N2 =m1g + m2g ➔


fk2 = μ2(m1g+m2g)
Magnitude of Friction – Kinetic Friction

Magnitude fk= μkN


Direction Opposite to Relative Velocity

a
v
m

μ
Magnitude of Friction – Kinetic Friction

Magnitude fk= μkN


Direction Opposite to Relative Velocity

N N = m (g + a) fk = μm(g + a)
a
v
fk m
μ
mg
Magnitude Of Friction – Static Friction

rest
F
m

μ
Magnitude Of Friction – Static Friction

Magnitude fs Varies from Situation to Situation

Direction fs max ≤ μsN


Opposite to tendency of Relative Motion

θ
Magnitude Of Friction – Static Friction
Magnitude fs Varies from Situation to Situation
fs max ≤ μsN
N Direction
Opposite to tendency of Relative Motion
rest
F
fs m

μ
fs

mg

F - fs = 0 ➔ fs = F ; Provided fs ≤ μN
θ

mg sin θ - fs = 0 ➔ fs = mg sin θ; Provided fs ≤


μN
Magnitude Of Friction – Static Friction
rest
F
F
m

θ
Magnitude Of Friction – Static Friction

fs
rest
F F
m
N

θ
mg

mg - fs = 0 ➔ fs = mg ; Provided fs ≤ μN F - mg sin θ - fs = 0 ➔ fs = F – mg sin θ;


Provided fs ≤ μN
Magnitude Of Friction – Static Friction
Magnitude fs Varies from Situation to Situation
fs max ≤ μsN
Direction
Opposite to tendency of Relative
N Motion

a
Pseudo Force = ma m fs
a
No Slipping

. mg .

ma - fs = 0 ➔ fs = ma ; Provided fs ≤ μN
Magnitude Of Friction – Static Friction
Magnitude fs Varies from Situation to Situation
fs max ≤ μsN
Direction
Opposite to tendency of Relative
Motion

a
m
a
No Slipping

. .
Analysis Of Friction Shift from Static to Kinetic
F=t
10 kg

μs = 1.0
μk = 0.8
Example Identify the type and magnitude of friction f v/s t Graph & a v/s t Graph.

F = αt
m

μs , μk
F = αt
m

μs , μk
Identify the type and magnitude of friction f v/s t Graph and a v/s t
Example
Graph.
F = αt
m
Solution:
Initially body will be at rest till the time some min value of force is
μs , μk
attained

Body at Rest
fs = F fs = αt
fs ≤ μsN αt ≤ μsN t ≤ μsN/α
Static Friction
Body just at the verge of motion
fs = F fs = αt
fs = μsN αt = t = μsN/α
μ sN
Body just after motion starts
Kinetic Friction
fk = μkN t > μsN/α
Example Identify the type and magnitude of friction f vs t Graph and a vs t
Graph F = αt
m
Solution: μs , μ k

Initially body will be at rest till the time some min value of
force is attained

Body at Rest f
fs = F fs = αt static kinetic
fs ≤ μsN αt ≤ μsN t ≤ μsN/α
μsN
Body just at the verge of motion
μkN
fs = F fs = αt
fs = μsN αt = μsN t = μsN/α t
μsN
Body just after motion starts
α
fk =
t > μsN/α
μkN
Example Identify the type and magnitude of friction f vs t Graph and a vs t
Graph. F = αt
m μs , μ k
Solution:
Initially body will be at rest till the time some min value of force is attained

Static Friction
Body at Rest 0 ≤ t ≤ μsN/α
a=0
a=0
Body just after motion starts t = (μsN/α)+
fk = μkN
( μs − μk ) N Kinetic Friction
α ( μsN/α ) − μkN
a= =
m m αt − fk
a=
Body after motion starts t > μsN/α m
αt − μkN
a=
m
Example Identify the type and magnitude of friction f vs t Graph and a vs t
Graph. F = αt
m μs , μ k
Solution:
Initially body will be at rest till the time some min value of force is attained

Body at Rest 0 ≤ t ≤ μsN/α


a=0 a
Body just after motion starts t = (μsN/α)+
static kinetic
fk = μkN
α ( μsN/α ) − μkN ( μs − μk ) N
a= = m
m ( μs − μk ) N
Body after motion starts t > μsN/α m
αt − μkN t
a=
m μsN
α
f a

static kinetic static kinetic

μsN
( μs − μk ) N
μkN
m
t t
μsN μsN
α α
Friction
LECTURE 2
Angle of Repose
Angle of Friction

θ
Magnitude of Friction
Types of Friction

Static Friction Kinetic Friction


f a
static kinetic static kinetic

μsN
( μ s − μk ) N
μkN
m

t t
μsN μsN
α N α

F= αt
f μ
Analysis Of Friction Shift from Static to Kinetic
Draw f v/s t Graph & a v/s t Graph
F=t
10 kg

μs = 1.0
F f
μk = 0.8

t t
a

t
Example Identify the type and magnitude of friction f v/s t Graph & a v/s t Graph.

F = αt
m

μs
μk F f

t t
a

t
F = αt
m

μs f
μk

t
a

t
Identify the type and magnitude of friction f v/s t Graph and a v/s t
Example
Graph.
F = αt
m
Solution:
Initially body will be at rest till the time some min value of force is
μs , μk
attained

Body at Rest
fs = F fs = αt
fs ≤ μsN αt ≤ μsN t ≤ μsN/α
Static Friction
Body just at the verge of motion
fs = F fs = αt
fs = μsN αt = μsN t = μsN/α

Body just after motion starts


Kinetic Friction
fk = μkN t > μsN/α
Example Identify the type and magnitude of friction f vs t Graph and a vs t
Graph F = αt
m
Solution: μs , μ k

Initially body will be at rest till the time some min value of
force is attained

Body at Rest f
fs = F fs = αt static kinetic
fs ≤ μsN αt ≤ μsN t ≤ μsN/α
μsN
Body just at the verge of motion
μkN
fs = F fs = αt
fs = μsN αt = μsN t = μsN/α t
μsN
Body just after motion starts
α
fk = μkN t > μsN/α
Example Identify the type and magnitude of friction f vs t Graph and a vs t
Graph. F = αt
m μs , μ k
Solution:
Initially body will be at rest till the time some min value of force is attained

Static Friction
Body at Rest 0 ≤ t ≤ μsN/α
a=0
a=0
Body just after motion starts t = (μsN/α)+
fk = μkN
( μs − μk ) N Kinetic Friction
α ( μsN/α ) − μkN
a= =
m m αt − fk
a=
Body after motion starts t > μsN/α m
αt − μkN
a=
m
Example Identify the type and magnitude of friction f vs t Graph and a vs t
Graph. F = αt
m μs , μ k
Solution:
Initially body will be at rest till the time some min value of force is attained

Body at Rest 0 ≤ t ≤ μsN/α


a=0 a
Body just after motion starts t = (μsN/α)+
static kinetic
fk = μkN
α ( μsN/α ) − μkN ( μs − μk ) N
a= = m
m ( μs − μ k ) N
Body after motion starts t > μsN/α m
αt − μkN t
a=
m μsN
α
f a

static kinetic static kinetic

μsN
( μs − μk ) N
μkN
m
t t
μsN μsN
α α
Angle of Repose

θ
Measuring Coefficient of Friction practically
➔ Place a body of mass m on a rough horizontal surface
(angle of surface with horizontal is adjustable)

The body will remain stationary

➔ Now, gradually start increasing the angle of inclination (θ) of the surface.
Angle of Repose - Calculate

θ
Angle of Friction

f
μ
Angle of Friction (α)
➔ It is the angle made by resultant of contact forces with normal reaction
➔ Consider a body on a rough surface with normal reaction (N) & frictional
force (f) acting on it.

N
R
m

f
Problems on Static Friction

Static, Kinetic Friction

0 < fs < μsN

fk = μkN
Example Angle of inclination θ of plane is double the angle of repose. The angle of
repose is α = 300. Find the acceleration of a block sliding down this plane.
g = 10 m/s2

A. 5/√3 ms-2
B. 10/√3 ms-2
C. 10√3 ms-2
D. 10 ms–2 α

θ
Angle of inclination of plane is double the angle of repose. The angle of
Example
repose is α = 300. Find the acceleration of a block sliding down this plane.
g = 10 m/s2
Solution:

Angle of inclination, θ = 2 α
N
we know, a f
1
μ = tan α= tan 300 = mg sin θ
√3
mg sin θ – f = ma mg cos θ
θ
N = mg cos θ
f = μ N = μ mg cos θ
Solution:

So,
mg sin θ – μ mg cos θ = ma

a = g sin θ – μ g cos θ

1
= 10 sin 600 – cos 600
√3
= 10
√3 –
1
×
1
2 √3 2

10 m/s2
a=
√3
A force of 20 N can just move a 3 kg block kept on a horizontal ground.
Example
Calculate the coefficient of friction & angle of friction. (g = 10m/s2)

3 Kg 20 N
A force of 20 N can just move a block kept on a horizontal ground. The
Example
mass of the block is 3 kg. Calculate the coefficient of friction & angle of
friction. (g = 10m/s2)

Solution: N

m = 3 kg
F = 20 N
F = fL = 20N
fL

From FBD, mg = 30 N
fL = μN = 20 Newton
N = mg = 30 Newton
Solution:
fL 20 2 R N
μ = = =
N 30 3

angle of friction : α F = 20 N
fL
2 fL
tan α = =
N 3
mg = 30 N
2
α = tan–1
3
Example Find the minimum value of m1 for which system remains in equilibrium.

A. m2/μ m1
B. m2μ
C. 2m2/μ μ
D. √m2μ

m2
Example For the given arrangement, find the minimum value of mass m1 for which
the system remains in equilibrium. Assume, pulley & strings to be
massless.
Solution:
➔ For system to be equilibrium, block m1 must not move. N T
➔ The minimum value of mass m1 has to be in co-relation
with limiting friction acting on it.
➔ FBD of system
m1 T m2
ƒ

m1g m2g
Solution:

For equilibrium
T = m2g , N = m1g
N T
ƒS = T ≤ μN
So,
m 2g ≤ μ N m1 T m2
m 2g ≤ μ m 1 g ƒ

μ m 1 ≥ m2 m1g m2g

m1 m2

μ
m2
∴ (m1) min. =
μ
An object of mass M = 10 kg is kept on a rough table as seen from above.
Example Forces are applied on it as shown. Find the direction of static friction if
the object does not move. (Take μ = 0.4)
A. 37° with 20 N force
B. 37° with 15 N force
C. 143° with 15 N force
D. 143° with 20 N force

10kg 20 N

15 N
In figure below, an object of mass M = 10 kg is kept on a rough table as
Example seen from above. Forces are applied on it as shown. Find the direction of
static friction if the object does not move. (Take μ = 0.4)
Solution: N
The FBD of the block will be:
Here limiting value of friction force:
flim = μrN = 0.4 × 10 × 10 = 40 N

The resultant of external forces acting on the block Mg

Fnet = (15)2` + (20)2 = 25N

20
If the block is at rest, f = Fnet = 25 N N

(Since, f < flim) Fnet


15 N

Here actual friction force acting on the block is less than


flim & the friction in this case is of static nature.
Solution:
fx
For the direction of friction force, we draw the
θ 20 N
free body diagram and find the resultant force.
The direction of static friction is opposite to the θ
Fnet
direction of the resultant force Fnet. 15 N

Its magnitude is equal to 25 N at angle:

15
tan θ = = 3
20 4

⇒ θ = 37°

Then, from the figure, fx makes an


angle 143° with the 20 N force.
Assuming the block of mass of m2 to be moving downwards and m1 to be
Example
moving upwards, on the given inclined plane of coefficient of friction μ,
Find the acceleration of two blocks.

α β
ALTERNATIVE

α β
Example Assuming the block of mass of m2 to be moving downwards and m1 to be
moving upwards, on the given inclined plane of coefficient of friction μ,
find the acceleration of two blocks.

Solution:
N1 a
For body 1 T

N1 = m1g cosα

T – m1g sinα – ƒ1 = m1a m1g sinα


f1
Also, m1g cosα

ƒ1 = μ N1 = μm1g cosα

∴ T – m1g sinα–μm1g cosα = m1a . . . (i)


Solution:

For body 2

N2 = m2g cosβ

m2g sinβ – ƒ2 – T = m2a


m2g sinβ
Also, m2g cos β

ƒ2 = μN2 = μm2g cosβ

∴ m2g sinβ – μm2g cosβ – T = m2a . . . (ii)


Solution:
Now, T – m1g sinα – μm1g cosα = m1a . . . (i)

m2g sinβ – μm2g cosβ – T = m2a. . . (ii)


+ + + +
m2g sinβ – m1g sinα – μm2g cosβ – μm1g cosα = (m1+ m2)a

m2g (sinβ – μcosβ) – m1g (sinα + μcosα) = (m1+ m2)a

m2(sinβ – μcosβ) – m1(sinα + μcosα)


a= g
m1 + m2
Example A block of mass 15 kg is just resting on a rough inclined plane as shown.
The block is tied by a horizontal string which has a tension of 50N . Find
the coefficient of friction between the surfaces of contact. (g = 10m/s2)
A. 0.3
B. 0.7
C. 0.6
D. 0.5

45°
45°
Solution:

Let's draw the free body diagram

N
N T cos450
450
T
fL
fL
T sin450

mg sin450
mg cos450
450 Mg
Solution:
For equilibrium
N T cos450
(Mg + T)
N = Mg cos450+ T sin450 =
√2
fL
Mg sin450 =T cos450 + fL
(Mg – T)
ƒL = Mg sin450 – T cos450= T sin450
√2
Also, mg sin450
μ (Mg + T) mg cos450
ƒL = μN =
√2
So, μ (Mg +T ) (Mg – T )
=
√2 √2
1
μ =
Mg – T 150 - 50 100 2
μ= = =
Mg +T 150 + 50 200
The friction coefficient between the board and the floor shown in figure
Example
below is μ. Find the maximum force that the man can exert on the rope so
that the board does not slip on the floor.

μ
Solution: (M+ m)g
T
T
T
T
T
T
Let’s draw the FBD of all the bodies: F
T
Let F is the force applied by man on rope. Hence, f
T T
tension in string, T = F. N

The man is in equilibrium in vertical direction

Along vertical direction,

∑ Fy = 0:
⇒ N + T = (M + m) g
⇒ N = (M + m)g − T

If horizontal direction of the board is not sliding on


floor, then f = T and friction should be static in
nature of f ≤ fmax.
Solution:

⇒ T = m[(M + m)g − T]

⇒ T(1+m) = m(M + m)g

μ (M + m)g
⇒T ≤
1+μ

For maximum value of T, we have:

μ (M + m)g
T =
1+μ
Friction
LECTURE 3
2 Block Problems
Example Find the Range where the mass can be kept without slipping! The value of R ?
A. 1m
B. 1.5m
C. 1.6m
D. 1.8m

37o
37 37
o o
2 Block Problems

μ
Introduction to 2 Block System

F F
Smooth m1 Smooth m1
Smooth m2 Rough m2

m1 moves and m2 stays at rest

m1 Smooth m1
Smooth F
F m2
Smooth m2 Rough

m2 moves and m1 stays at rest


Introduction to 2 block system
F
Rough m1
Smooth m2

F m1 F
m1
m2 m2

Both move separately


Both move together
Static friction Kinetic friction
2 Block System – Both Moving Together

μ m1 F F
a =
m2 m1 + m 2

a
m1 F
f

F – f = m 1a f ≤ fL

Fm2 Fm2
⇒ f= ⇒ ≤ μm1g
m1 + m 2 m1 + m 2
m1(m1 + m2)
For static friction ⇒ F ≤ μ g
m2
2 Block System – Both Moving Separately

μ m1 F
m2

a1
a2
N1
m1 F f
f m2

m1g N
m2g N2
1
F – μN 1 =m1a1 f = m2a2
F – μm1g μm1g
⇒ a1 = a2 =
m1 m2
For kinetic friction a1 ≥ a 2
Analysis of 2 Block System

μ
Analysis of Two block system F
Rough m1
Smooth m2
Step 1 : Assume any 1 scenario either static
friction or kinetic friction
Step 2 : Apply condition for static friction or
condition for kinetic friction
Step 3 : Solve by taking other scenario if
conditions for 1st scenario are not satisfied
Solution:

➔ For system to be equilibrium , block m1 must not move.


➔ The minimum value of mass m1 has to be in co-relation with limiting
friction acting on it.
➔ FBD of system
N T

m1 T m2
ƒ

m 1g m2g
Example Find acceleration of 4 kg & 6 kg in the given scenario.

A. 5 ms–2
μ=1 4 kg 50 N
B. 2 ms–2
C. 8.33 ms–2 6 kg

D. 1 ms–2 smooth
μ=1 4 kg 50 N ALTERNATIVE
6 kg

smooth
Example Find acceleration of 4 kg & 6 kg in the given scenario.

μ=1 4 kg 50 N
Solution:
6 kg

Step 1 : Assuming 4 kg and 6 kg move together smooth


50 =
a = 5 ms–2
4+6 a = 5 ms–2
50 – f = 4(5)
⇒ f = 30 N FBD of 4 kg 4 kg 50 N
f
40 N
Step 2 : Check f ≤ fL
fL = μN = 1(40) = 40

Clearly f ≤ fL as 30 < 40
Example Find friction acting on 10 kg block.
A. 30 N
μ = 0.6 5 kg 30 N
B. 20 N 10 kg
C. 12 N
smooth
D. 9 N
Example Find friction acting on 10 kg block

μ = 0.6 5 kg 30 N
Solution:
10 kg

Step 1 : Assuming 5 kg and 10 kg move separately smooth

FBD of 5 kg a1 FBD of 10 kg a
2

5 kg 30 N N
f1 = μN
f1 = μN 10kg
50 N
10g N
30 – 0.6(50) = 5a1
0.6(50)1 = 10a2
a1 = 0 m/s2 a = 3 m/s2
2
Step 2 : Check a1 ≥ a2 as friction is kinetic
0 <3 ∴ this assumption is not correct
Solution:

Step 3 : Solve assuming both move together μ = 0.6 5kg


30 N

10 kg
FBD of 10 kg
N a = 2 ms–2 smooth
10kg f

10g N1 F = 10(2) = 20 N

Checking f ≤ fL
fL = μN = 0.60(50) = 30

Clearly 20 < 30

∴ This assumption is correct


Example Find acceleration of 5 kg block.

5kg
μ = 0.3
A. 0.6 ms–2 15kg 40N
B. 5 ms–2
smooth
C. 2 ms–2
D. 4.8 ms–2
Example Find acceleration of 5 kg block.
μ = 0.3 5kg
Solution: 15kg 40 N

smooth
Step 1 : Assuming both move together
40
a = = 2 ms–2
5 + 15
40 – f = 15(2) FBD of 15 kg a = 2 ms–2
N
⇒ f = 10 N 40 N
f 15 kg
Step 2 : Check f ≤ fL
fL = μN = (0.3)(50) = 15 50 N1

Clearly f ≤ fL
Example Find acceleration of 2 kg block.
F = 18 N
μ = 0.5 2 kg
A. 2.7 ms–2
4 kg
B. 1.5 ms–2
smooth
C. 3 ms–2
D. 4 ms–2
Example Find acceleration of 2 kg block. F = 18 N
μ = 0.5 2 kg

4 kg
Solution:
smooth
Step 1 : Assuming both move together
18
a = = 3 ms–2
4+2 FBD of 4 kg a = 3 ms–2
f = 4(3) = 12 N
f
4kg
Step 2 : Check f ≤ fL
fL = μN = (0.5)(20) = 10 N
Clearly f ≤ fL ∴ assumption is incorrect
Step 3 : Both move separately FBD of 2 kg a1
18 – (0.5)(20) = 2a1
fk = 0.5(20) 2 kg 18
a1 = 4 ms–2
Example Find acceleration of 10 kg block .

A. 2 ms–2
10 kg
B. 5 ms–2 F = 100 N
μ = 0.5 40 kg
C. 1 ms–2
D. 4 ms–2
Smooth
Example Find acceleration of 10 kg block .

Solution: μ = 0.5 10 kg F = 100 N


40 kg
Step 1 : Both move together
100 Smooth
a= = 2 ms–2
40 + 10

f = 10(2) = 20 N FBD of 10 kg a = 2 ms–2

fL = μN = (0.5)(100)= 50 N 10 kg
f
Step 2 : Check f ≤ fL
50 N1
Clearly f ≤ fL as 20 < 50
The blocks M and m are arranged as the situation in figure below. The
Example
coefficient of friction 2 blocks is μ1 and that between the bigger block and
the ground is μ2. Find the acceleration of the block M.

μ1

m
M

μ2
μ1
M m
m
M

μ2
Solution:

FBD of mass M: FBD of mass m:


aM am = aM
x
T T
1
T N1 amy
f1 M
N1 μ1 m
Mg f1 mg
N2
T
f2
μ2

T(aM) + T (aM) - T (amy) = 0 ➔ 2(aM) = ( amy)


Solution:

For the motion of block m:


Along horizontal direction:
N1 = ma …(i)
a
Along vertical direction: T
mg − (μ1 N1+ T ) = m(2a) N1 2a
f1
After substituting value of N1 in Eq. (i), we have mg
mg − (μ1 ma + T)= m (2a)
…(ii)
Solution:
For the motion of block M:
Along vertical direction:
∑ FV = 0
…(iii)
⇒ N2 = T + μ1 N1 + Mg
a
Along horizontal direction:
2T − (N1 + μ2N2) = Ma …(iv)
T
T
From equation (i) and (iii), we get:
N1
2T − [N1 + μ2 (T + μ1 N1 + Mg)] = Ma Mg f1
N2
⇒ 2T − [ma + μ2 T + μ1μ2 (ma)+ μ2 Mg] = Ma …(v) T
f2
Now solving equations (ii) and (iv), we get:
[2m− μ2 (M + m) ] g
a=
M + m[5 + 2(μ1− μ2)]
Friction

DPP 1
A block of mass 2kg is placed on the floor. The coefficient of static
friction is 0.4. If a force of F is 2.5N is applied on the block, the frictional
force between the block and the floor will be

A. 2.5 N F = 2.5 N
B. 5N 2kg
C. 7.8 N
𝛍 = 0.4
D. 8N
2 persons, pull each other through a massless rope in 'tug of war' game.
Who will win?
A. One whose weight is more
B. One who pulls the rope with a greater force
C. One who applies more friction force (shear force) on ground
D. One who applies more normal force (compressive force) on ground
A block of mass 4 kg is kept on ground the coefficient of the friction between the
block and the ground is 0.80. An external force of magnitude 30N is applied
parallel to the ground. The resultant force exerted by the ground on the block is ?

A. 40N
B. 30 N
F = 30N
C. 32 N 4kg
D. 50 N
𝛍 = 0.8
A rough vertical board has an acceleration 'a' so that a block pressing against it
doesn’t fall .The coefficient of friction b/w the block & board is ________
A. >g/a
B. <g/a
C. =g/a 𝝁

D. >a/g
m
ALTERNATIVE

m
A bead of mass m is located on a parabolic wire with its axis vertical and vertex
directed towards downwards as in figure and whose equation is
x2 = ay. If the coefficient of friction is 𝝁, the highest distance above the x-axis at
which the particle will be in equilibrium is

A. 𝝁a y
B. 𝝁2a
C. 𝝁2a/4
x2 = ay 𝛍
D. 2𝝁a
m

O x
Figure shows a block kept on a rough inclined plane. The maximum external force
down the incline for which the incline for which the block remains at rest is 1N
while the maximum external force up the incline for which the block is at rest is
7N. The coefficient of static friction 𝛍 is :

A.

B.

30° 30°
C.

D.
A solid cube of mass 5kg is placed on rough horizontal surface, in xy - plane as
shown. The friction coefficient between the surface and the cube is 0.4. An
^ N is applied on the cube. (use g = 10 m/s2)
external force F = 6i^ + 8j^ + 20k

A. The block starts slipping over the surface Multi-Correct

B. The friction force on the cube by the surface is 10N

C. The friction force acts in xy - plane at angle 127° with the positive x-axis in clockwise direction

D. The contact force exerted by the surface on the cube is 10√10 N.


z

𝛍 = 0.4
x
A solid cube of mass 5kg is placed on rough horizontal surface, in xy - plane as
shown. The friction coefficient between the surface and the cube is 0.4. An
^ N is applied on the cube. (use g = 10 m/s2)
external force F = 6i^ + 8j^ + 20k

A. The block starts slipping over the surface Multi-Correct

B. The friction force on the cube by the surface is 10N

C. The friction force acts in xy - plane at angle 127° with the positive x-axis in clockwise direction
D. The contact force exerted by the surface on the cube is 10√10 N.
z

𝛍 = 0.4
x
Friction

DPP 2

θ
A 60 kg body is pushed horizontally with just enough force to start it moving across a
floor and the same force continues to act afterwards. The coefficient of static and
sliding friction are 0.5 & 0.4 respectively. The acceleration of the body?

A. 6 m/s2
B. 4.9 m/s2
C. 3.92 m/s2
60 kg
D. 1 m/s2
𝛍s = 0.5
𝛍k = 0.4
A block of mass M = 5 kg is resting on a rough horizontal surface for which the
coefficient of friction is 0.5. When a force F = 50N is applied, the acceleration of the
block will be (g = 10 m/s2)

A. 6.0 m/s2
F = 50N
B. 8.0 m/s2
C. 3.17 m/s2
37°
D. 10.0 m/s2 M
𝛍 = 0.5
The block moves towards right under action of horizontal force F(figure a). Sometime
later another force P is applied to the block making an angle θ (such that tanθ = μ)
with vertical as shown in fig b. After application of force P, the acceleration of block
shall
P
(b)
(a)
𝜽

F F
m m

𝛍 𝛍
(a) (b)
𝜽
F F
m m

𝛍 𝛍
A force F= 2t (where t is time in seconds) is applied at t=0sec to the block of mass m
placed on a rough horizontal surface. The coefficient of static and kinetic friction between
the block and surface are μs and μk respectively. Which of the following graphs best
represents the acceleration vs time of the block (μs >μk) ?

A.
a
t

B.
a
t

C.
a
t

D.
a
t
f a
static kinetic static kinetic

μsN
( μs − μk ) N
μkN
m

t t
μsN μs N
α N α

F= αt
f μ
An insect of mass m, starts moving on a rough inclined surface from point A. As the
surface is very sticky, the coefficient of friction between the insect and the incline is
μ=1. Assume that it can move in any direction, up or down the incline then

A. The maximum possible acceleration of the insect can be 14m/s2 Multi-Correct


B. The maximum possible acceleration of the insect can be 2m/s2
C. The insect can move with a constant velocity
D. The insect cannot move with a constant velocity

𝜽 = 37°
An insect of mass m, starts moving on a rough inclined surface from point A.
As the surface is very sticky, the coefficient of friction between the insect and
the incline is μ=1. Assume that it can move in any direction, up the incline or
down the incline then Multi-Correct

A. The maximum possible acceleration of the insect can be 14m/s2


B. The maximum possible acceleration of the insect can be 2m/s2
C. The insect can move with a constant velocity
D. The insect cannot move with a constant velocity
A

𝛍=1

𝜽 = 37°
A chain of length L is placed on a horizontal surface. At any instant x is the
length of chain on rough surface and the remaining portion lies on smooth
surface. Initially x = 0. A horizontal force F is applied to the chain. In the
duration x changes from x = 0 to x = L, for chain to move with constant speed.
A. the magnitude of F should increase with time
B. the magnitude of F should decrease with time
C. the magnitude of F should increase first and then decrease with time
D. the magnitude of F should decrease first and then increase with time

L
x
F

smooth rough
A block of mass 5 kg & surface area 2m2 just begins to slide down an inclined
plane when the angle of inclination is 30°. Keeping mass same, the surface area
of the block is doubled. The angle at which this starts sliding down is :

A. 30o
B. 60o
C. 15o
D. 45o
A plank of mass 8 kg with block of 2 kg placed on its rough surface, lie on a
smooth floor of elevator ascending with an acceleration g/4. The coefficient of
friction is μ = 1/5 between the block & plank. A horizontal force F = 30 N is
applied to the plank. Then the acceleration of block & plank w.r.t. elevator are:
A. 3.5 m/s2, 5 m/s2 g/4
B. 5 m/s2, 50/8 m/s2
C. 2.5 m/s2, 25/8 m/s2
D. 4.5 m/s2, 4.5 m/s2

μ = 1/5 2 kg
F = 30 N
8 kg
g/4

μ = 1/5 2 kg F = 30 N
8 kg
A worker wishes to pile a cone of sand into a circular area in his yard. The
radius of the circle is r, and no sand is to spill onto the surrounding area. If μ is
the static coefficient of friction between each layer of sand along the slope and
the sand, the greatest volume of sand that can be stored in this manner is :
A. μπr3
B. 1/3 μπr3
C. 2 μπr3
D. 3 μπr3
h

θ
r
A side view of a simplified form of vertical latch B is as shown. The lower
member A can be pushed forward in its horizontal channel. The sides of the
channels are smooth, but at the interfaces of A and B, which are at 45° with the
horizontal, there exists a static coefficient of friction μ = 0.4. What is the
minimum force F that must be applied horizontally to A to start motion of the
latch B if it has a mass m = 0.6 kg ?
A. 7N
B. 10 N
C. 12 N
D. 14 N
B

F
A
B

F
A
Frictional force of 2 kg is f1 & on 3 kg is f2 and T is tension in the string
which of the following is correct.
y
A. f1 = 8î, f2 = -15î, T = 2N
B. f1 = -8î, f2 = -15î, T = 2N x
C. f1 = 5î, f2 = -15î, T = 5N
D. f1 = -5î, f2 = -15î, T = 5N
T
2 kg 3 kg

μ = 0.4 μ = 0.5

1 kg
2 kg
T
2 kg 3 kg

μ = 0.4 μ = 0.5

1 kg
2 kg
A smooth pulley Po of mass 2 kg is lying on a smooth table. A light string passes
round the pulley and has mass 1 kg & 3 kg attached to its ends. The 2 portions
of the string being perpendicular to the edge of the table so that the masses
hang vertically. Pulley P1 & P2 are ideal [g = 10 m/s2]
COMPREHENSION
1. Tension in the string is -
A. 12 N
B. 6N
C. 24 N Po
D. 18 N
P1
2 kg
P2

1 kg

A
B
Po

P1
2 kg
P2

1 kg

B
A smooth pulley Po of mass 2 kg is lying on a smooth table. A light string passes
round the pulley and has mass 1 kg & 3 kg attached to its ends. The 2 portions
of the string being perpendicular to the edge of the table so that the masses
hang vertically. Pulley P1 & P2 are ideal [g = 10 m/s2]
COMPREHENSION
2. Acceleration of Pulley Po is -
A. 2 m/s2
B. 4 m/s2
C. 3 m/s2 Po
D. 6 m/s2
P1
2 kg
P2

1 kg

A
B
A smooth pulley Po of mass 2 kg is lying on a smooth table. A light string passes
round the pulley and has mass 1 kg & 3 kg attached to its ends. The 2 portions
of the string being perpendicular to the edge of the table so that the masses
hang vertically. Pulley P1 & P2 are ideal [g = 10 m/s2]
COMPREHENSION
3. Acceleration of Block A is -

A. 6 m/s2
B. 4 m/s2
C. 3 m/s2 Po
D. 8 m/s2
P1
2 kg
P2

1 kg

A
B
A 1 kg block B rests on a bracket A of same mass as shown in figure. Constant
forces F1 = 20 N and F2 = 8 N start acting at time t = 0. The distance of block B
from pulley is 50 m at t = 0. Determine the time (in s) when block B reaches the
pulley.
A. 5s
B. 4s 50 m
F2 = 8 N
C. 3s
D. 8s
B
F1 = 20 N
A

You might also like